Wikipedia:Reference desk/Science: Difference between revisions

From Wikipedia, the free encyclopedia
Content deleted Content added
Line 594: Line 594:
::Names like that are often taken from a person. I don't know who "Manav" would be, though. [[User:Looie496|Looie496]] ([[User talk:Looie496|talk]]) 14:16, 19 October 2012 (UTC)
::Names like that are often taken from a person. I don't know who "Manav" would be, though. [[User:Looie496|Looie496]] ([[User talk:Looie496|talk]]) 14:16, 19 October 2012 (UTC)
:::Yep, I thought of that too. Problem is that there appears to be no trace of anyone named "Manav" or "Manavo" who is related to Madagascar or bats. [[User:Leptictidium|Leptictidium]] (''[[User talk:Leptictidium|mt]]'') 14:50, 19 October 2012 (UTC)
:::Yep, I thought of that too. Problem is that there appears to be no trace of anyone named "Manav" or "Manavo" who is related to Madagascar or bats. [[User:Leptictidium|Leptictidium]] (''[[User talk:Leptictidium|mt]]'') 14:50, 19 October 2012 (UTC)
::::According to [http://malagasyworld.org/bins/teny2/manavy this page] (bottom entry), there's a Malagasy word ''manavy'' that denotes a small bat. ''Manavi'' is likely a Latinate representation of that. [[User:Deor|Deor]] ([[User talk:Deor|talk]]) 18:07, 19 October 2012 (UTC)


== About [[Karl Emil Lischke]] ==
== About [[Karl Emil Lischke]] ==

Revision as of 18:07, 19 October 2012

Welcome to the science section
of the Wikipedia reference desk.
Select a section:
Want a faster answer?

Main page: Help searching Wikipedia

   

How can I get my question answered?

  • Select the section of the desk that best fits the general topic of your question (see the navigation column to the right).
  • Post your question to only one section, providing a short header that gives the topic of your question.
  • Type '~~~~' (that is, four tilde characters) at the end – this signs and dates your contribution so we know who wrote what and when.
  • Don't post personal contact information – it will be removed. Any answers will be provided here.
  • Please be as specific as possible, and include all relevant context – the usefulness of answers may depend on the context.
  • Note:
    • We don't answer (and may remove) questions that require medical diagnosis or legal advice.
    • We don't answer requests for opinions, predictions or debate.
    • We don't do your homework for you, though we'll help you past the stuck point.
    • We don't conduct original research or provide a free source of ideas, but we'll help you find information you need.



How do I answer a question?

Main page: Wikipedia:Reference desk/Guidelines

  • The best answers address the question directly, and back up facts with wikilinks and links to sources. Do not edit others' comments and do not give any medical or legal advice.
See also:



October 12

Dot product and cross product

We know "Work = Force x Displacement", where "Work" is scalar quantity and both "Force" and "Displacement" are vector quantity. This means dot product of vectors is scalar quantity. We also know that cross product of vectors is a vector quantity. Suppose, we are given "Power = Force x velocity". Here, both "Force" and "Velocity" are vector quantity and we have to find whether "Power" is scalar or vector quantity. It becomes easy if we have idea that "Force x velocity" is dot product or cross product. My question is how to recognize the given product of vectors is dot product or cross product. Please, also give me some examples where cross product of vectors is a vector quantity. Sunny Singh (DAV) (talk) 12:28, 12 October 2012 (UTC)[reply]

Any quantity is a vector if it is directionally dependent. Does a concept like "power" or "work" have a directional component? If so, it is a vector. If the value doesn't depend on the direction, it is a scalar. --Jayron32 12:44, 12 October 2012 (UTC)[reply]
Question, is there a difference between simple multiplication and the cross product? Plasmic Physics (talk) 12:51, 12 October 2012 (UTC)[reply]
Multiplication of vectors, Cross product. TenOfAllTrades(talk) 13:01, 12 October 2012 (UTC)[reply]

We don't just arbitrarily multiply vectors and then try to figure out what formula to use. We use mathematical formalism to make a simple calculation about a physical quantity we care about. A dot product of two vectors represents the physically meaningful concept of projection. Loosely stated, this calculation measures how "similar" two vectors are; or how closely aligned they are; and it is scaled by the magnitude of each vector. We can also normalize if we are concerned only with geometry, and not magnitude. We often use the scalar result of a dot product to scale another vector, if that represents some physical, useful quantity. The cross-product is a little more unusual, because its physical interpretation is somewhat less intuitive; but simply put, a cross product guarantees orthogonality. There are many situations in physics where that property of vectors has physical meaning - like when we're calculating properties in rotating reference frames; or calculating interactions with magnetic fields, spinning objects, and fluid or particle ensembles. My point is, you decide which calculation you need entirely based on the physics; not just based on arbitrary combinations of your input variables. Nimur (talk) 14:25, 12 October 2012 (UTC)[reply]

Nimur's point is what I was going after, but in less eloquent terms as he puts it. Mathematics is a tool in these cases used to elucidate the physics, not the other way around. The physics of a situation drives what mathemetics we use to help explain it. If a physical quantity has a direction, vector mathematics is used to describe it. If a physical quantity is directionless, scalar mathematics is used to describe it. The question first to be asked when trying to decide whether a quantity is scalar or vector should be "does this physical property depend on direction." --Jayron32 16:13, 12 October 2012 (UTC)[reply]

Torque is an example where the cross product is used. The result of the applied force will be a rotation around an axis perpendicular to the force and the lever. Lorentz force is another example: the force on a charged particle in a magnetic field is perpendicular to the direction of the field and to the direction it's moving in (the velocity vector). For a charged particle in an electric field on the other hand, the force will be in the direction of the field. Angular velocity can also be represented as a vector, and the cross product is used here too. One way to decide whether dot or cross product should be used is considering two cases, one with both vectors in the same direction, the other with perpendicular directions. If the result should be maximum for the first and zero for the second, then you would probably use the dot product; in the opposite case, the cross product. Ssscienccce (talk) 10:55, 13 October 2012 (UTC)[reply]

Two questions for project

Does anyone know what structural settings bending folds are most likely to be found in? What about buckling folds? This will be useful in identifying where different structures formed. Thanks.~~--- Anon — Precedingunsigned comment added by 99.146.124.35 (talk) 12:46, 12 October 2012 (UTC)[reply]

In what context, geology? Plasmic Physics (talk) 12:49, 12 October 2012 (UTC)[reply]
Yes structural geology. — Preceding unsigned comment added by99.146.124.35 (talk) 13:08, 12 October 2012 (UTC)[reply]
List of orogenies? See also Fold (geology).--Shantavira|feed me 16:12, 12 October 2012 (UTC)[reply]

Malaria in Europe

From my previous question I realized that malaria was present in Europe some centuries ago, but now it has been eradicated. How could be possible that malaria mosquitoes specifically disappear but that you still get mosquitoes in Europe? What's the difference between one kind and the other?Gorgeop (talk) 14:12, 12 October 2012 (UTC)[reply]

Is genetic resistance to malaria perhaps what you are looking for? Mosquitos are not born with malaria, they need to ingest it from an infected host to be able to pass it on. If a strong resistance to malaria removes the disease from the population, then no amount of vectors will matter since there is no disease to pass on. Livewireo (talk) 14:42, 12 October 2012 (UTC)[reply]
Malaria eradication was very much intentional. We have an article National Malaria Eradication Program for the U.S. - not sure if we have the equivalent for Europe. History of malaria makes an extraordinary claim indeed, that Anopheles gambiae got loose in South America, caused the worst malaria epidemic ever seen in the New World, but was then completely exterminated by eradication efforts within a few years. Which, I have to say, seems more ambitious than programs against invasive species tend to be today, even if they had to use Paris Green (and ordinaryPyrethrum) to do it according to our article - and it was done in northeast Brazil! (Apparently this was just before DDT was discovered to be an insecticide ... not sure if it played a role; certainly it did in other efforts) Of course, those were also the days when "draining the pestilential swamps" was seen as a positive thing. Wnt (talk) 14:59, 12 October 2012 (UTC)[reply]
Generally speaking the major way to eliminate malaria was to drastically decrease mosquito populations (vector control). Historically this was achieved by drainage and later pesticides. By driving down the number of mosquitos for an extended period of time, you can decrease the amount of overall malaria in the ecosystem to basically zero levels, because the malaria parasite cannot reproduce without mosquitos. You will never really get rid of all of the mosquitos, and their populations will rebound, over time, but without the malaria parasites. Mosquito monitoring and control is no doubt still in effect (in the United States, the Centers for Disease Control and Prevention does this); if all mosquito control efforts stopped tomorrow, over time malaria would likely return to Europe and other places it has been "eradicated" from. --Mr.98 (talk) 14:59, 12 October 2012 (UTC)[reply]
See The history of malaria in England which says that locally transmitted malaria was finally eradicated from the UK in the 1950s, which is surprisingly late. We still have mosquitoes though. Alansplodge (talk) 18:03, 12 October 2012 (UTC)[reply]
Malaria is transmitted by Anopheles gambiae mosquitos and these are not like other mosquitos that are still extant in Europe. It's easier to eradicate this species, since they are not able to "hibernate", therefore, you just have to hit them hard once during the winter, when they are more prone to be eradicated. The eggs that they lay won't be dormant for months and hatch when it's hot again. Summary: it was easier to get rid from malaria transmitting mosquitos in Europe, but others are more resilient. OsmanRF34 (talk) 19:52, 12 October 2012 (UTC)[reply]
The parasite needs certain species of mosquitoes to reproduce and human or animal hosts to spread. Mosquitoes don't live long and will feed only a few times before they die. For the parasite to spread, you need infected persons (or other secondary hosts if the mosquito is a species that feeds on animals as well) in whom the parasite has had time to infect the blood cells, a mosquito of the right species has to bite him and get infected, the parasite needs time to reproduce in the mosquito and then the mosquito has to feed on another person before it dies. Infected people will eventually be diagnosed and treated, so when the infection rate is low, the number of hosts will decrease steadily. Ssscienccce (talk) 12:05, 13 October 2012 (UTC)[reply]

Volume of naphtha

How many British Thermal Units of naphtha can fit into one cubic foot or cubic meter? I have a figure with a specific number of BTUs, and I'm trying to figure out its total volume, so I went to Wolfram Alpha, but all I got was "BTUIT (IT British thermal units) and ft3 (cubic feet) are not compatible." 2001:18E8:2:1020:749C:5B76:1D8E:3D22 (talk) 14:24, 12 October 2012 (UTC)[reply]

Well our article has a density down as "750-785 kg/m3" - with that you can work from a measure of BTU per kg, though actually a direct statement of BTU per volume might be more accurate. (This won't be perfectly accurate in any case because with that much of a density range there must be some variation in composition) Wnt (talk) 15:12, 12 October 2012 (UTC)[reply]
... and the top Google hit yields:[1]
  • Petroleum naphtha "lower heating value" - 116,920 btu/gal; 19,320 btu/lb; 44.938 MJ/kg;
  • Petroleum naphtha "higher heating value" - 125,080 btu/gal; 20,669 btu/lb; 48.075 MJ/kg; density 2,745 g/ft3
  • NG-based FT naphtha "lower heating value" - 111,520 btu/gal; 19,081 btu/lb; 44.383 MJ/kg;
  • NG-based FT naphtha "higher heating value" - 119,740 btu/gal; 20,488 btu/lb; 47.654 MJ/kg; density 2,651
"[1] The lower heating value (also known as net calorific value) of a fuel is defined as the amount of heat released by combusting a specified quantity (initially at 25°C) and returning the temperature of the combustion products to 150°C, which assumes the latent heat of vaporization of water in the reaction products is not recovered. The LHV are the useful calorific values in boiler combustion plants and are frequently used in Europe.
"The higher heating value (also known as gross calorific value or gross energy) of a fuel is defined as the amount of heat released by a specified quantity (initially at 25°C) once it is combusted and the products have returned to a temperature of 25°C, which takes into account the latent heat of vaporization of water in the combustion products. The HHV are derived only under laboratory conditions, and are frequently used in the US for solid fuels."
I'll leave it to you to double check all these numbers agree with each other. Wnt (talk) 15:20, 12 October 2012 (UTC)[reply]
Engineering and technical books usually distinguish between the LHV and HHV, as, usually but not always, the LHV is what matters to an engineer. However, it is common in college science texts to just quote a single value without stating which it is, and when they do that, it's the HHV, as it is the HHV that is measured in a simple bomb calorimeter. Ratbone124.178.45.41 (talk) 15:58, 12 October 2012 (UTC)[reply]
Actually, for the HHV / LHV question, what matters depends on the context. As an engineer who deals with this on a regular basis, HHV is what matters because that is the basis on which the fuel is supplied. It can be confusing when equipment suppliers quote performance on a LHV basis, as this gives higher efficiency numbers. --Pakaraki (talk) 06:38, 22 October 2012 (UTC)[reply]
Note that the way you asked this Q implies that a BTU is a unit of volume. Instead, I suggest that this would be a clearer Q: "How many British Thermal Units can be generated by burning the naphtha which can fit into one cubic foot or cubic meter ?". StuRat (talk) 18:45, 12 October 2012 (UTC)[reply]
Hmm, perhaps I misunderstood what a BTU is. I thought it was the amount of gas (or whatever other fuel) needed to produce a certain amount of thermal energy? 2001:18E8:2:1020:749C:5B76:1D8E:3D22 (talk) 20:41, 12 October 2012 (UTC)[reply]
Actually, a British Thermal Unit is a unit of energy. Devices having nothing to do with flammable materials, like an electrical air conditioner, are also rated in BTUs: [2]. StuRat (talk) 21:14, 12 October 2012 (UTC)[reply]

DUTASTERIDE/ASPARTAME combined side effects

many articles give detailed side effects of DUTASTERIDE and many articles give detailed side effects of ASPARTAME, but no article gives effect of both these taken together. If an article could be given on this topic of a pointer to the sites where this could be found would be good. Thanks — Preceding unsigned comment added by117.223.102.73 (talk) 16:25, 12 October 2012 (UTC)[reply]

I added a title. StuRat (talk) 18:42, 12 October 2012 (UTC) [reply]
Unless this has been studied to see if the combined side effects are worse than just the total of each, there is unlikely to be any such article. And, unfortunately, the number of combinations of two substances is too large for every combo to be studied. StuRat (talk) 19:04, 12 October 2012 (UTC)[reply]
  • Don't hold your breath for such an article when this search provides zero results. -- Scray (talk) 19:59, 12 October 2012 (UTC)[reply]
dutasteride is a 5-alpha-reductase inhibitor, one could check if these enzymes are involved in aspartame metabolism. But that still wouldn't tell you much. Ssscienccce (talk) 13:21, 13 October 2012 (UTC)[reply]
Indeed you have discovered the Achilles heel of "safe chemicals" and drugs in general. One may be reliably safe in tests, but two in combination can be a different matter. The classic example of this is melamine, which is pretty much safe to contaminate/adulterate foods with ... until it meets up with cyanuric acid to form big plates of melamine cyanurate that concentrate and clog up the kidneys.
That said, the odds of any given interaction are, well, "probably very low". And because dutasteride resembles a steroid and aspartameresembles a small peptide, their interactions shouldn't be all that different from interactions of other chemicals within the body ... unless they are. Wnt (talk) 19:51, 13 October 2012 (UTC)[reply]
Well said. Chance of any interaction is low, but polypharmacy is a big challenge, and the number of potential interactionsgrows exponentially with the number of medications (our article makes this statement, and the math's pretty simple, but it would be nice to find a reliable source that shows this). Food for thought. -- Scray (talk) 20:13, 13 October 2012 (UTC)[reply]

evolution

Is it possible that someone studying our civilization 30,000 years from now differentiates caucasoid, mongoloid, and negroid the same way we differentiate neanderthal, sapien and denisavan? As different species due to the relatively narrow geneome that would exist at the future time?165.212.189.187 (talk) 18:51, 12 October 2012 (UTC)[reply]

I doubt it. A species is defined in sexually reproducing organisms as any group which can reproduce to produce fertile offspring. Clearly this is the case with all humans (even extinct Neanderthals). StuRat (talk) 18:55, 12 October 2012 (UTC)[reply]
No. While the members of a race have certain genetic similarities not shared with other races, they do not form monophyletic groups and cannot be classed as subspecies. See race and genetics for more information. Someguy1221 (talk) 18:56, 12 October 2012 (UTC)[reply]
Yes, those skeleton types are easily distinguishable, as are others. The "there is no race" POV is based on cultural and ethical, not skeletal arguments. μηδείς (talk) 20:00, 12 October 2012 (UTC)[reply]
I can differentiate tall people from short people based on skeletal morphometry, too (and height has a substantial genetic component); however, the question was about species. Differences in skeletal morphometry do not define species. -- Scray (talk) 20:13, 12 October 2012 (UTC)[reply]
Indeed. Of course there is such a thing as race. But it is not a biologically well-defined concept. I think Medeis has it backwards. The ways that we define races are cultural, and not scientific. The scientific perspective is that the concept of human races (as culturally defined) is not biologically useful or meaningful (e.g. as Someguy1221 describes), not that "there is no race." See Race_(human_classification)SemanticMantis (talk) 20:27, 12 October 2012 (UTC)[reply]
The question is whether different races could be considered to be different species, not whether different races exist. StuRat(talk) 20:19, 12 October 2012 (UTC)[reply]
I don't think the OP is arguing in a technical sense that they are separate species, he seems to be using the terms loosely. It is unlikely that future scientists would describe the races as species, since they would see unmistakable signs of interbreeding and intergradation. But the current general types will still be distinguishable to them in skeletons of our age. The differences are much more distinct than just height, white people aren't just black people with light skin. Scientists can and do distinguish negroid type skeletons from caucasoids and mongoloids and so forth all the time. Distinctions politicians and layman make may or may not correspond. Whatever it is that semanticmantis wants exactly to deny it is up to him to describe clearly. Just to say biologist, anthropologists, and other scientists don't make distinctions of race is either false, overly-vague, or a wish. μηδείς (talk) 01:47, 13 October 2012 (UTC)[reply]
Medeis, no is arguing that you can't classify people as being members of a race, nor does interbreeding and integration have anything to do with it. Rather, race simply has no meaning in cladistics, except as a geo-phenotypically defined paraphyletic grouping. Identifying a person as mongoloid carries about as much taxonomic information as pointing out that a dog is black and fluffy. Someguy1221 (talk) 02:00, 13 October 2012 (UTC)[reply]
Well, seriously, no. Telling me someone is a caucasoid tells me a lot more about what to expect about his skeleton and other statistically likely things than does telling me he's black and fluffy. Perhaps the term terrier would be more closely analogous to a "race" than black and fluffy? As for cladistics not applying to races, that will be a surprise to population geneticists and linguistic classifiers[3] for example. I am not interested in this debate, denying the reality of race is a facile way of claiming scientific sophistication. What is important is what is affirmed, not what is denied. We will see if the OP clarifies his statement.μηδείς (talk) 02:19, 13 October 2012 (UTC)[reply]
The graph doesn't refute either of us. You can take literally any group of living things and construct a phylogeny. I don't know how they constructed theirs, so I can't comment on it. I'm not saying that scientists have not worked with racial classifications in phylogeny, I'm trying to say it's not useful. In layman's terms, the group of people described as "mongoloid" does not include all living descendants of the group's most recent common ancestor, by far. In other words, there are people who genetically belong to the mongoloid classification as much as anyone who is phenotypically mongoloid does. There are even people who are phenotypically mongoloid who genetically fit in better with some other racial classification. So going back to my very first comment, the races are paraphyletic groups. If you knew any taxonomy aside from what you randomly googled, you'd know that no taxonomist would ever intentionally construct a paraphyletic group. For the most part, they only exist because many classifications were conceived before sequencing existed (some newer ones exist because species were ordered in a phylogeny before detailed sequencing was done at all). But I will give you one thing, I went too far when I said race is meaningless to cladistics. It has meaning, but I maintain that "mongoloid" is as much a species/subspecies/any-monophyletic-group as "black fluffy dog". Someguy1221 (talk) 03:38, 13 October 2012 (UTC)[reply]
It's also true that there are monophyletic groups that almost contain all of a given race. This is why cladograms like the one you showed, as well as the ones in our own article on the subject, are so easy to make. Someguy1221 (talk) 03:41, 13 October 2012 (UTC)[reply]
  • Have a look at this [4] recent assessment of human phylogeny. As depicted in the first figure, human races started varying around, oh, 150,000 years ago. That's a long, long time, but it's nothing compared to the 500,000 years separating us from Neanderthals. Remember - Neanderthals persisted until very recent times (relative to that) so they were a new species. Of course, long ago, there was a time when sapiens and neanderthal were brothers in the same family, and there's a time when any two brothers today, or any two races, could be the prototype of a new species. But the races need not be the prototypes for the split - it is just as possible that a single mixed-race population subsequently divides (the nerds of all races at your local high school launch off into space...). Wnt (talk) 22:57, 12 October 2012 (UTC)[reply]
  • Actually the biggest distinction among current humans is between Bushmen/Pygmies and everybody else. But even there the level of genetic similarity is much higher than between Neanderthals and Modern Man. Regarding Denisova Man, all we have from them is exactly one finger bone, one toe bone, and one tooth, so pretty much everything we can say about them comes from genetics. Looie496 (talk) 23:02, 12 October 2012 (UTC)[reply]
Despite all the pointless arguing, nobody has hit the main point. No, future scientists will not classify today's humans as separate species, because what counts as a species isn't defined based on currently-living humans. The most common definition of "species" is "a group of organisms capable of interbreeding and producing fertile offspring." There is no question that today's humans can interbreed and produce fertile offspring. Therefore, unless scientists decide to redefine species, today's humans are and forever will be the same species. --140.180.242.9 (talk) 10:38, 13 October 2012 (UTC)[reply]
Well, you usually think of the last common ancestor, and if you send some sample humans off in a flying saucer, that last common ancestor has already been born. True, it's very very unlikely, but we can't rule it out.
I am also rather curious about how much interbreeding actually occurs between the Twa or other "pygmies" and the rest of humanity. I've entertained the sci-fi speculation that these small, efficient humans await merely the development of interstellar colonization and 20 generations or so of strong selection to reveal themselves as a successor species to H. sapiens. Wnt (talk) 00:43, 15 October 2012 (UTC)[reply]

I thought Neanderthals and Sapiens could produce offspring?165.212.189.187 (talk) 12:51, 15 October 2012 (UTC)[reply]

The biological species concept is a really fuzzy concept. It's not always the case that a "mule" is sterile. For example, mallards are famous for breeding with any duck they can meet up with, and "contaminating" species in this way. There's even the odd case of ring species, which will freely interbreed along a continuum of terrain ... but act as different species at the ends! At a rough approximation, species are groups of animals that not merely can but do interbreed, or would interbreed... maybe someone else understands the philosophy here more than I do, because at some point I tend to think of the distinction as more semantic and arbitrary than deeply meaningful. I suspect for example that the incredibly rapid African cichlid adaptive radiation events have something to do with the ancestral population being made up of hybrids to begin with.
When it comes to Neanderthals, the record doesn't look like free and open mating between the two populations, but rather some very limited transfers. For example, if you backcross offspring of a rare fertile female mule back into the population, you can transfer some horse genes to the ass or vice versa (I don't know if it works both ways offhand), but it doesn't make the two the same species. Wnt (talk) 19:52, 15 October 2012 (UTC)[reply]

Couldn't that just be a result of proximity/ logistics?GeeBIGS (talk) 23:23, 16 October 2012 (UTC)[reply]

Even to the point: Can a Newfoundland breed with a Yorkshire Terrier? Would that make Dogs a "ring species"? --Jayron32 19:55, 15 October 2012 (UTC)[reply]

Sooo.... then it is "possible" or what???165.212.189.187 (talk) 14:44, 16 October 2012 (UTC)[reply]

Anything is possible, but this is extremely unlikely. StuRat (talk) 02:53, 17 October 2012 (UTC)[reply]


October 13

Are they possible? Difficult to implement? Expensive? How big? OsmanRF34 (talk) 00:02, 13 October 2012 (UTC)[reply]

Yes. No. No. Small. Zoonoses (talk) 03:23, 13 October 2012 (UTC)[reply]
  • I'm no expert, but a little searching provided some references that address your questions: [5][6] [7] -- Scray (talk) 12:12, 13 October 2012 (UTC)[reply]

Physical process at work when airing out some smokey clothes?

I went to a social event last night that featured a great deal of smoking. Not a smoker, I was annoyed that my suit reeked of cigarettes when I got home. Without giving it much consideration, I hung my suit outside to "air out" overnight. Then I realized I have no idea what is literally happening during this process, meanwhile the outcome is assumed. What happens to the cigarette scent? The particulates don't just magically jump off my suit, do they? Why does it freshen? The Masked Booby (talk) 01:08, 13 October 2012 (UTC)[reply]

Liquid components can evaporate, solids can either sublimate or be blown off. UV light from the sun may also cause chemical reactions, and other scents (like in pollen) may be deposited on the clothes and then disguise the smoke smell.StuRat (talk) 01:14, 13 October 2012 (UTC)[reply]
The word you are looking for is adsorption, the process by which gases and liquids come to cling to surfaces. There is a thermodynamic equilibrium between adsorption sites and adsorbents in the air. In smoky air, the amount attached to clothing will increase until the adsorption sites are more or less saturated. In clean air, the adsorption sites will slowly and spontaneously release the molecules they have been holding. This causes a lingering odor even after the clothing is removed from the smoky environment. The odor dissipates once all the attached molecules have released from the clothing and had a chance to drift away. When possible, washing the clothing is also generally effective at removing adsorbed odors. Dragons flight (talk) 05:06, 13 October 2012 (UTC)[reply]
Also notice that the reason to hang it outside isn't just to deodorize it, but also to prevent the odor from being deposited on other objects inside.StuRat (talk) 06:30, 13 October 2012 (UTC)[reply]

Why inorganic and + (plus sign)

I am confused why we classify CO and CO2 as inorganic compound even when both contain carbon. My another question is - While reading newspapers I always see + {sign} and some small colorful circles at the bottom of each page, where there is no text. I think these + and small circles have some importance, but I don't know what is that importance. Sunny Singh (DAV) (talk) 09:02, 13 October 2012 (UTC)[reply]

Both CO (carbon monoxide) and CO2 (carbon dioxide) are commonly created by inorganic processes, such as volcanism.StuRat (talk) 09:35, 13 October 2012 (UTC)[reply]
CO2 is also commonly created by organic processes, such as eukaryotic respiration. If you have a room full of chemical scientists, you'll be assured to have more definitions of the term organic than there are people in the room. Plasmic Physics (talk) 12:12, 13 October 2012 (UTC)[reply]
Personally, I prefer to define "organic molecules" as a molecule that contains both hydrogen and carbon, without exception. Plasmic Physics (talk) 12:15, 13 October 2012 (UTC)[reply]
That's probably too rigid. I don't think that any chemist I know would classify Mirex as "inorganic", and certainly PTFE would be counted as organic as well, despite not having any hydrogens. - As Graeme points out below, the reason for the organic/inorganic split is the initial belief invitalism, where the distinction between organic and inorganic wasn't based on atom composition, but on the (erroneous) belief that the materials that made up life had some "élan vital" that made them different from inorganic ones. The "contains carbon" was a post-hoc rationalization for the split. - Personally, my definition of "organic" is more along the lines of "contains carbon, except for a few historical exceptions". The "contains both hydrogen and carbon" might be okay if you altered it to "contains either carbon-hydrogen or carbon-carbon bonds". (As Plasmic Physics says: more definitions than people in the room). -- 205.175.124.30 (talk) 21:14, 13 October 2012 (UTC)[reply]
I know it's just an opinion, but based on vitalism, it seems silly that as a synthetic polymer applied to frying pans, PTFE is considered organic by most chemists. Likewise, Mirex just doesn't exactly scream organic either. Plasmic Physics (talk) 00:02, 14 October 2012 (UTC)[reply]

question two Printing registration --Digrpat (talk) 09:30, 13 October 2012 (UTC)[reply]

I added a title, but what is the question ? StuRat (talk) 09:34, 13 October 2012 (UTC)[reply]
I removed your heading, StuRat. Digrpat was answering the second question asked by Sunny Singh— the "+ {sign} and some small colorful circles" are registration marks. Deor (talk) 10:14, 13 October 2012 (UTC)[reply]
OK, thanks, I thought it was a new, misplaced Q. StuRat (talk) 10:24, 13 October 2012 (UTC) [reply]
The organic substances were originally though to be made only by living things, and there were other carbon containing moleculed like hydrogen cyanide or ammonium carbonate or calcium carbide that were classed as inorganic too. Eventually people figured out how to make organicurea and the distinction was blurred. Graeme Bartlett (talk) 12:14, 13 October 2012 (UTC)[reply]
  • I've seen various arbitrary definitions of organic compounds, such as "anything with carbon" or "anything with carbon and hydrogen". It looks like the definition can be different in practice, as I infer from [8], which defines volatileorganic compounds as being, well, volatile, and "any compound of carbon, excluding carbon monoxide, carbon dioxide, carbonic acid, metallic carbides or carbonates, and ammonium carbonate." I note that this excludes C, CO, CO2, H2CO3, Na2C2, Na2CO3, (NH4)2CO3. So the EPA is excluding even some things with hydrogen, so long as it has an "ionic" character to it, or is part of an ion; but they're not excluding, say, carbon tetrachloride orCFCs without hydrogen. So it's apparently a ... flexible ... definition. Wnt (talk) 20:27, 15 October 2012 (UTC)[reply]

Visual imagination of birth-blind people

Redirect me if this question was asked earlier, but can birth-blind people approximately imagine their daily surroundings, such as apartment they live in? Also, how much correct is a blind person's imagination, can (s)he describe for example a beautiful woman based on clues of his/her life experience?--176.241.247.17 (talk) 09:05, 13 October 2012 (UTC)[reply]

People who are congenitally blind have special skills in mentally-mapping spatial relationships they determine by feel PMID 17368576 and PMID 8124943, but some imagery functions are impaired PMID 11496156 and they don't construct detailed visual mental images in the same way that sighted people do (which can be a hindrance or an advantage, depending on the situation) PMID 16556565. "Beautiful", of course, is at least as much cultural as it is biological. -- Scray (talk) 11:48, 13 October 2012 (UTC)[reply]

Getting an oversized Disk though a circular hole in a bendable material ....

If you have a solid disk (eg like a CD) and you want to get it through a circular hole (eg to get it into a sealed plastic bag) (The plastic is bendable but not stretchable, the disk is rigid) you dont have to have the hole diameter as large as the disk. I know I have tried it.

If you bend the plastic along the diameter of the hole and flex it you can pass through through the hole the larger size diameter disk.

If the whole was infinitely 'flexible' (like a circular piece of string) it could be pulled to make a slit; then, if the circular hole had diameter 'd', clearly the disk could be oversized to a diameter d x pi/2 (approx 1.5d).

But the paper or plastic cannot be moved to give a slit shape... Probably the maximum opening depends upon the 'flexibility of the material with the hole cut into it...

... is there any maths of written in for on this!!??

(The problem has practical purpose as I want to get a flange through a hole into a sealed plastic gas collection bag, link this internal flange to an external flange fitted with a tube to give a gas tight seal - to get a tube for my gas collection bag to get it fit for purpose!)

Thanks email redacted — Preceding unsigned comment added by78.145.76.1 (talk) 11:08, 13 October 2012 (UTC)[reply]

If the opening of the container is flexible, then we could think about the problem as the circumference of the cross-section of the curved disc (you describe it as rigid, but it can be flexed), which would have as upper bound the cross-section of the flat disc (if d is diameter of the disc and x is the thickness, it would be 2d+2x, but x << d so it's approximately 2d). Curving the disc will, conceptually, shorten one side. Assuming that the disc cannot be curled to overlap, the minimum would be d (with the disc curled around to make a cylinder; obviously, the latter isn't easy with a CD). So, I think the opening needed is bounded by d..~2d, and depends on the flexibility of the disc. -- Scray (talk) 11:35, 13 October 2012 (UTC) Heron is correct in following comment - I reversed the problem. -- Scray (talk) 03:19, 14 October 2012 (UTC)[reply]
Scray, I think you are confusing the flexible plastic of the bag with the rigid plastic of the disc. The disc is rigid. The only reason OP can't stretch the hole into an arbitrarily narrow slit is that the material has a finite Young's modulus that resists bending. If the material were perfectly flexible but inelastic, like very fine chain mail, then you could perform the feat. It only seems impossible because the force required increases to infinity as you approach the ideal slit shape - a bit like trying to straighten a clothes line by pulling on the ends. Maths can help you only by telling you how much force it would take to deform the sheet to a given slit shape. --Heron (talk) 15:40, 13 October 2012 (UTC)[reply]
Quite right - sorry. -- Scray (talk) 03:19, 14 October 2012 (UTC)[reply]
The material has to be stretchable to some degree for that to work. If it isn't stretchable at all, the only thing you can do with it is fold it, and that isn't helpful. Looie496 (talk) 15:28, 13 October 2012 (UTC) Gah, what was I thinking?Looie496 (talk) 02:59, 14 October 2012 (UTC)[reply]
Agreed, given the constraints of a disk that wont bend at all and plastic that won't stretch at all, the slit in the plastic would need to be as wide as the disk. As far as getting a gas-tight seal, you could try a resealable bag. Those aren't quite a perfect seal, so you might need to add some tape around the edges and corners. You could also use shrink-fit plastic, which shrinks when heated with a blow-dryer. But, again, you may need to use tape to seal up any leaks. And, of course, plastic bags won't hold much pressure, in any case, and will slowly leak small molecules, no matter how well sealed they are. StuRat (talk) 20:54, 13 October 2012 (UTC)[reply]
Hmmm, I'm not absolutely sure that you couldn't expand the slit at all at one point by some deformation of the plastic around the hole that did not involve stretching. That sounds like a non-trivial math problem to me... Wnt (talk) 23:44, 13 October 2012 (UTC)[reply]
Perhaps there's some difference in how we are using the term "stretching". All materials can undergo both elastic and plastic deformation, to some degree, except for a theoretically "perfectly rigid solid" (these don't exist in real life, but crystals come close). Elastic deformation is when a material deforms and then returns to it's original shape. Plastic deformation is when a material deforms but does not return to it's original shape. As the name implies, soft (thermo)plastics tend to have great deal of plastic deformation, but still do have some elastic deformation. I am calling both types of deformation "stretching". Perhaps the OP is not including elastic deformation. In this case, it would be possible to elastically deform some plastics enough to allow a CD to be inserted, and then have the slit reduce back down to the original size. If you make the bag out of an elastic material, you could use a much smaller slit. StuRat (talk) 00:12, 14 October 2012 (UTC)[reply]

Turret milling machine or Universal milling machine..?

Hi I am going to start a new workshop and want to buy milling machine but I want to know which machine will be suitable for me turret milling machine or universal milling machine. My use of machine will be producing flat surface on small jobs of normally 100 X 200 mm. — Preceding unsigned comment added by 14.140.235.82(talk) 14:02, 13 October 2012 (UTC)[reply]

What will best suit your needs depends on a multitude of factors that we cannot know. Budget? Is this for a new hobby or some sort of small business? What are you going to machine - mild steel, aluminium, special alloys? Plastic? To what accuracy? Do you intend to machine something occaisonally or all day long every day? One would guess from the question that you are a beginner/hobbyist. As such, rather than ask us, a better approach would be to get hold of one or two good trades or hobby level books on machine metal working. Even better, subscribe to journals such asModel Engineer's Workshop for a year or so - you'll learn a lot. Then, join a local hobby engineering group or make contact with an interested tradesman, perhaps someone retired. You question is the sort best dicussed at length with a knowledgable person face to face rather than posting a question to an internet forum. If the highest accuracy is not required, and you only want to do this occaisonally, a better first purchase would be a good pedestal drill. You can fit a low cost X-Y table and milling cutters on a pedestal drill and do quite usefull work at the fraction of the cost of a proper milling machine. However, if your ultimate aim is certainly to end up doing work of professional standard, you should select a machine on the basis of easy upgrade to CNC. Don't forget also, that flat surfaces can be done quickly and very satisfactorily on a lathe, and a lathe can do a lot that a milling machine cannot do. Thats's why there's a lot more hobbyists and small shops that own a lathe as their only machining tool than there are hobbyists and shops that own a milling machine. Are you going to work in plasic, or can the things you want to make be done in plastic? If so, 3D printing is now the way to go. Ratbone60.228.235.209 (talk) 15:24, 13 October 2012 (UTC)[reply]
For that matter, if you only want to make a flat surface in metals such as steel, aluminium, brass, once in a while, say once each few months or more, and you have a good degree of patience, you don't need any powered machine tool at all. The hand method suitable for a job 100 x 200 mm is as follows:-
1. Cut to a slight oversize as best you can with a hacksaw - use a new blade and take it with nice and slow long strokes without forcing it. You should get to within 0.5 to 1 mm of true.
2. Use a coarse double cut barstard file (steel) or dreadnought file (aluminium or brass) to bring the surface to within 0.1 to 0.15 mm, checking with a steel rule. People who have not been properly trained in filling find it difficult not to produce a curved surface - clamping a piece of 30 x 30 square steel tubing to the backside of the file helps - put some shims between the file and the tubing to convex the file surface slightly to compensate for the natural human tendency to curve the work surface the other way. If you find it hard not to curve the work you are probably applying too much force or you have the workpiece to high or too low for you. Finish up by drawfilling with a single cut mill file, being carefull to only "load" the file with fingers over the middle of the workpiece.
3. If greater flatness than +,- 0.1 mm is required, get a piece of float glass to use as a reference surface. Use a can of spaycan paint to apply a THIN coat to one side of the glass. Don't wait for it to dry. Rub the painted surface LIGHTLY on to the work surface. You'll now have paint on the high spots of your work surface. Now, use a hand scraper to scrape metal only where there is paint. Repeat as necessary until the paint evenly coats the work. The professional version of this is to paint the work with marking blue or a felt tip pen and then rub it with a reference flat surface, and scrape where there blue or felt tip has been rubbed off.
With care and patience, you can make a surface flat to an accuracy exceeding that of the best machines with this method. But you WILL need patience. When you feel yourself getting bored and tending to force the tools, take a break. See http://en.wikipedia.org/wiki/Hand_scraper. Ratbone121.221.218.240 (talk) 11:24, 14 October 2012 (UTC)[reply]
An important question is whether or not the OP needs his flat surface to be parallel to, at right angles to, or at some specified angle to, another flat surface on the workpiece. Right angles can be done accurately on the lathe with some care and a dial probe. Right and arbitary angles can be done easily on a turret milling machine. They can only be done by hand scraping if both a reference flat surface and a reference angle block is avalailable. Reference blocks are readily available for right angles but not arbitary angles. Floda124.178.37.144 (talk) 02:58, 15 October 2012 (UTC)[reply]

Europa from Io

Having read the articles about Extraterrestrial skies and Apparent retrograde motion, here's what I'm wondering: What would Europa's path look like for an observer sitting at Io's antijovian point? It will return to the same position after two Ionian days, and there will be some sort of retrograde motion, but apart from that, I can't really visualize it.

Many thanks :) JaneStillman (talk) 14:25, 13 October 2012 (UTC)[reply]

Retrograde motion only appears when you look at something closer to the center than you are -- not the case here. The motion is quite simple. Let's start with the two moons aligned: Europa appears then at the zenith. For about a day, Europa drops toward the horizon. For about two days it is absent. Then it appears on the opposite horizon, and takes another day to climb back to the zenith. (The motion of Io as seen from Europa is much more complicated.) Looie496 (talk) 15:41, 13 October 2012 (UTC)[reply]
Actually, as I think about it, the apparent motion of Io as seen from Europa is not complicated either. You simply see it revolving around the central planet which remains fixed in the sky -- the apparent speed of revolution is exactly half of the true speed. Looie496 (talk) 17:09, 13 October 2012 (UTC)[reply]
Could you clarify what you mean by "Retrograde motion only appears when you look at something closer to the center than you are", Looie. It's the planets that are farther from the Sun than Earth that appear to have periods of retrograde motion (from our perspective); Mercury and Venus don't.Deor (talk) 22:30, 13 October 2012 (UTC)[reply]
Yes they do — when they're on our side of the Sun. —Tamfang (talk) 05:26, 14 October 2012 (UTC)[reply]
My simulation gives a curve resembling a cardioid, with a small loop in place of the cusp. —Tamfang (talk) 05:26, 14 October 2012 (UTC)[reply]
I have to admit that the answer I gave was wrong, because I forgot about the fact that "retrograde" is defined with respect to the stars. On an ordinary satellite that's the only reasonable way to define it, but on a tidally locked satellite like Io or Europa it's a little strange. The fact is, as I said, that if you stand on Io and watch Europa, you will see it steadily rise in the sky and steadily fall, without ever reversing direction. But at the same time the stars also rise and fall. Most of the time Europa is rising or falling faster than the stars, but there is, I should have realized, a short period of time while Europa is near the zenith when the stars outpace it. And that, by the usual definition, is retrograde motion.Looie496 (talk) 14:56, 14 October 2012 (UTC)[reply]

Atomicity of elements

I want to know about elements in which more than 4 atoms combine to form molecules in elemental state apart from the following:

And is it true that osmium is octa-atomic and ditungsten and dimolybdenum is possible with sextuple bond

  • Well, we actually have a sextuple bond article, and it's totally news to me. The diagram in the article makes it look like 5 d orbitals and one s orbital are involved. Searching for "sd5 hybridization" and molybdenum turned up [9], which says it is found in HC(triple bond)MoH3; but I've never heard of sp3 hybridized carbon, say, using all four bonds to interact in a two-atom molecule ... but that's not really quite the same. Hmmm. It should be very interesting to hear a serious inorganic chemist comment on this one. Wnt (talk) 20:26, 13 October 2012 (UTC)[reply]
  • I'm thinking your list of molecules may be hard to define. For example, grab a lump of elemental metal, any metal, in vacuum. Ta-da, instant "molecule", with a bazillion atoms "bound together" to some degree or other. One can argue that fullerenes fall, very loosely, into this category.
  • Also, I'm not really aware of a [C=C=C=C=C=C=] ring (benzene contains hydrogen, of course). Diamonds are another type of very very large molecule. Wnt (talk) 20:30, 13 October 2012 (UTC)[reply]
Could you clarify the question, i.e. criteria for inclusion? For example, do you include Boron as you do because of Cs2B12H12? If so, that's not really in elemental form, right? Same goes for benzene, as Wnt asks. If you include these, then do all of the atoms of the same element need to be contiguous? I find the "elemental" version of the question interesting, but then the list you gave must be edited. -- Scray (talk) 20:41, 13 October 2012 (UTC)[reply]
Also, do they have to be neutral? If you type "boron clusters" into NCBI you'll get a lot of references; some discuss anions, however. See[10] and [11] for example. I get the feeling (not sure though) that you can pretty much blast boron into any size fragment you like; I suppose this is true of most elements, unless smaller compounds (like N2) are strongly preferred. Wnt (talk) 20:45, 13 October 2012 (UTC)[reply]
Do they have to be stable, or do you also consider at high pressure or other special conditions, high-energy states, and short-lived species? Nature is crazy...she gives us tetraoxygen and octaoxygen. DMacks (talk) 21:32, 13 October 2012 (UTC)[reply]
If you allow charge you can get Hydrogen ion clusters and Pentazenium. Also there are octaoxygen, tetraoxygen and fullerenes with less than 60 atoms and no charge. Sulfur can form all sorts of ring sizes apart from 8. Graeme Bartlett (talk) 21:42, 13 October 2012 (UTC)[reply]

With charge Pentazenium and others are radicals which I am not considering and my version of boron is boron allotrope gamma boron. I am not considering clusters inside compounds but have considered benzene ring because it can be considered a structural unit of graphene andgraphite. Thanks for mentioning tetraoxygen and octaoxygen and please mention other exotic ones.Solomon7968 (talk) 13:58, 14 October 2012 (UTC)[reply]

"The current price per package of tasonermin (Beromun 4 vials 1 mg) is EUR11,813.54. "

Come on! How can 4 mg cost that much? If they are protected by patents, is there a black market somewhere for this? It seems economically more attractive to produce such kind of things, being at 2.5 millions/gram than any other illegal drug. OsmanRF34 (talk) 22:33, 13 October 2012 (UTC)[reply]

Monopoly prices are absolutely arbitrary. (For a shining example of contemporary medical ethics, see Colcrys.) Nonetheless, four milligrams of highly purified, active human protein is not something that is really that easy to cook up in a lab. Note that a specific protein, unlike cocaine, isn't something that you can mix up with gasoline and salt and shake out of solution - you have to baby it through the isolation process to avoidprotein denaturation. A misfolded protein wouldn't work - I'm guessing it might even trigger some damaging autoimmune response. I'm not that well versed in business to pull out the commercial details from the company, but my feeling is that if you got a graduate student to whip something up in a week in a basement laboratory, it would cost a substantial fraction of the price, and many bad things could happen.
That said, I can picture alternatives and workarounds. Notably, TNF-alpha can be induced by things like gram (-) endotoxin[12], which is undoubtedly vastly cheaper and more available, though that of course can (will) induce other cytokines in a different pattern than the purified cytokine. From [13] note also that diet affects TNF-alpha production - more omega-3 fatty acids lead to less production in blisters. (I suspect that "dry" cupping therapy is a practice which will be reexamined in the near future, for this and some other reasons e.g. thymosin - the fluid produced by blisters is a who's who of potentially therapeutic proteins, though in other cases they are undesirable. Maybe that's nuts, but then again, I was the only person I knew in the 90s who was soft on Lamarckism :) ) But there are many, many ways to get TNF-alpha production to increase, and I probably haven't thought of the best here. My guess, in short, is that a black market in purified TNF-alpha is not very feasible. Wnt (talk) 23:24, 13 October 2012 (UTC)[reply]
That sounds about right, actually. If I want to buy 4 mg of recombinant human TNF for the lab tomorrow, that'll cost me £7200 from Peprotech (my usual cytokine supplier). And that's research quality, which didn't need to be qualified for clinical use. In this case, there isn't even a patent on it, although there probably is one on tasonermin specifically with regards to formulation etc etc. Recombinant proteins are expensive any way you look at it. As for using something else to increase endogenous TNF production, good luck in finding something that specifically upregulates TNF production without changing any other cytokine levels. (Hint: don't bother, it doesn't exist) Fgf10 (talk) 08:56, 14 October 2012 (UTC)[reply]
OK, but along the production line, where does the money goes to? In this case, no patent, as stated above, so? Machines? Technical personal? Long cooking process? Necessary extraction from humans? OsmanRF34 (talk) 21:41, 14 October 2012 (UTC)[reply]
Ooh, now that's tricky question! First off, it's not extracted from humans (haven't checked this specific one, but in general). These sort of things are made by inserting the human TNF gene in a bacterium or yeast, which is then grown in massive bioreactors. It won't be a very labour intensive process, so that probably won't be the main expense. The consumables for purifying the protein from the bioreactor will be a big chuck of the cost. The thing to remember as well is that 4 mg might seem like next to nothing, but in the world of recombinant proteins, it's an absolutely massive amount. For research use, your normal order would be in the order of 10 to 100 ug.
Of course overhead and near-monopoly pricing has a lot to do with the final price as well. To be honest, I doubt anybody who doesn't work for a biotech company who makes stuff like this can give a sensible answer here (and they wouldn't of course. I will note however, that prices (outside clinical use at least) are flexible, for instance, the £7200 I quoted further up is the list price, for such a bulk order I would expect to actually pay between a quarter or half of that price. Sorry for not being able to give a more conclusive answer. Fgf10 (talk) 07:15, 15 October 2012 (UTC)[reply]
Agreed. Another component of the pharmaceutical price has to be maintenance of a liability fund - litigation with biologics is pretty common and expensive in some markets. You mentioned the costs of production for clinical use: GMP. -- Scray(talk) 08:56, 15 October 2012 (UTC)[reply]
I should revise my statement above to some degree - it might be a bit more feasible... also less so. The similarity in price between lab TNF and the pharmaceutical seemed suspicious ... looking up the Beromun tasonermin, I find that it is actually a non-glycosylated recombinant protein from E. coli, a trimer of three peptides - it's not being produced in the fancy way that nature does it, i.e. TNF that's been inserted into the membrane, covalently modified, and chopped free by an enzyme.[14] It turns out that the human TNF doesn't actually have any glycosylation site. [15] Being grown in bacteria makes it cheaper, and potentially easier to pirate than I'd thought, though still by no means easy. The amino acid sequence used matches aa 77-233 of the human reference sequence [16] -VRSSSRTPSDKPVAHVVANPQAEGQLQWLNRRANALLANGVELRDNQLVVPSEGLYLIYSQVLFKGQGCPSTHVLLTH TISRIAVSYQTKVNLLSAIKSPCQRETPEGAEAKPWYEPIYLGGVFQLEKGDRLSAEINRPDYLDFAESGQVYFGIIAL - that's the same C-terminus at the end, and the same ADAM17 site as in the normal human form. I note that there's a different site at 79 used by MDC9 in vivo,[17] but I haven't looked up if that's a significant amount in the natural version (and it might not have the desired drug activity).
The other problem is that "The clinical indication for Beromun is - as an adjunct to surgery for subsequent removal of the tumour so as to delay or prevent amputation, or in the palliative situation, for irresectable soft tissue sarcoma of the limbs, used in combination with melphalan via mild hyperthermic isolated limb perfusion (ILP)." Maybe it has other indications now, or someone has an off-label use for it, but obviously, you'll have a hard time selling it to the sort of "specialised centres" that they say should be performing the procedure, and if somehow you do, well... the patients will never know about it. I wonder if people who actually care about a stray $15000 are really going to get this treatment anyway...
This approach more generally is based on Coley's toxins, a practice from the 19th century which still fires the imagination. There is such an array of potential cytokine stimulants, tumor antigens, and adjuvants that could interact to enhance an immune response - and immunology remains such a dark art - that to this day the approach offers people at least small doses of hope. Wnt (talk) 13:54, 15 October 2012 (UTC)[reply]
Well, the structure of the recombinant sounds pretty similar to the native protein, that's a homotrimer as well. The fact that's it's not cleaved from the membrane doesn't really matter, as that's not the active form anyway. The homology of the used protein is pretty good, and it will obviously be active in humans, or it would never have come anywhere near the market. The relevant datasheet[18] from the European Medicines agency, (which Wnt already linked to), actually describes the manufacturing process relatively detailed, and it's easy to see where the money goes, as it's pretty complicated. As for their indication, (without reading through the whole thing, admittedly) I can see it working in some tumours (those expressing high levels of TNFR1), but it's a very blunt approach to say the least. Certainly not going to be mild on the side-effects. On an entirely unrelated note, I was just doing some Western blotting on Wnt signalling components today, amussing to see Wnt commenting here as well.... Fgf10 (talk) 15:39, 15 October 2012 (UTC)[reply]


October 14

Hypnosis and alcohol blackouts

Can hypnosis aid in recovery of the memories lost to a blackout drunk? What I've read suggests that they're never properly stored, thus leaving nothing to recall in the first place, but has anyone compared the accuracy of recovered memory to impartial evidence (recordings, etc.)? And if so, what did they find? 71.248.115.187 (talk) 00:29, 14 October 2012 (UTC)[reply]

It's really unlikely that there would be any data specifically addressing that question, because I can't imagine that an Institutional review board would ever approve a study that involved inducing blackouts in subjects. That being said, this is an area where I'm sort of an expert, and my best guess is that there would never be any substantial recall, but it's possible that elements of the experience could be brought back by providing the person with cues that are associated with them. That's what often happens when you deal with people with amnesia, anyway. I doubt that hypnosis would be helpful. It could easily make the person invent pseudo-memories and believe that they are real memories, but if you are looking for genuine memories, those pseudo-memories would just get in the way. Looie496 (talk) 02:37, 14 October 2012 (UTC)[reply]
You're an expert on alcohol-induced amnesia ? Might I suggest Alcoholics Anonymous ? :-) StuRat (talk) 02:48, 14 October 2012 (UTC) [reply]
Recovered-memory therapy is probably quite relevant. Just btw, there are many different study designs which would not necessarily need to "induce" a black out. Case-control study, prospective and Retrospective cohort study could all gather completely relevant data specifically addressing this question. Louie's referring no doubt to the "gold standard" of trials, a double blind Randomized controlled trial, which I agree would probably be tricky to get passed by a ethics review board. And I completely agree with his conclusion, sounds highly implausible. Hypnotism is a widely misunderstood topic. Vespine (talk) 21:46, 14 October 2012 (UTC)[reply]

Does Capsaicin harm plants or their roots?

I have some houseplants I like to keep outside until first frost, but I have a problem with squirrels digging in their pots to the point of entirely uprooting some. I have tried capsaicin in small amounts before (cryy powder) but this wasn't enough to deter them. I am thinking of going all out with some chopped jalapenos or mustard powder on the soil, but am afraid I may hurt the roots. There are a few anecedotes on the web and a high school student experiment with sunflowers that say there will be no effect on the plant. Any good sources that will confirm this, especially on the roots? Thanks. μηδείς (talk) 16:49, 14 October 2012 (UTC)[reply]

Won't birds just eat the jalapenos ? Sean.hoyland - talk 16:57, 14 October 2012 (UTC)[reply]
I was going to chop them up and spread them in surface of the soil. μηδείς (talk) 17:00, 14 October 2012 (UTC)[reply]
Lots of links at Google "Capsaicin effect on plant roots". Never mind, of course you've already done that :-) hydnjo (talk) 19:27, 14 October 2012 (UTC) c/e strike by hydnjo (talk) 19:49, 14 October 2012 (UTC)[reply]
Actually, that was a good suggestion, and I did find the first link very helpful. μηδείς (talk) 20:00, 14 October 2012 (UTC)[reply]
Well I'll be darned for outguessing you/me outguessing me/you! hydnjo (talk) 20:28, 14 October 2012 (UTC) [reply]
Guess I'll bring them inside now, thanks. μηδείς (talk) 19:33, 14 October 2012 (UTC)[reply]
As far as next year, you might want to consider a physical barrier to the squirrels. One method is to hang them from trees using a long wire. Some type of cage around pots on the ground might also work (perhaps a birdcage) ? StuRat (talk) 19:49, 14 October 2012 (UTC)[reply]
Well, I like your idea of hanging the squirrels from trees using a long wire. But a cage around the plants somewhat defeats the purpose. (They arepoinsettias I keep over the summer and reflower each year.) μηδείς (talk) 19:58, 14 October 2012 (UTC)[reply]
LOL. But, seriously, bird cages can be attractive. StuRat (talk) 20:54, 14 October 2012 (UTC)[reply]
These plants are at a relative's in NJ, the idea of cages wouldn't go over too well, although a small cage with a much smaller-leafed and spreading flowering plant might actually be attractive. I'll post a picture of my favorite after it begins to flower. The main problem is that these are the only potted plants in that yard that don't spread out over the soil (they have a small tree-like growth form) and with the bare soil they are the only ones the squirrels pick on. So far no disasters this year, but in the past they have knocked them over and split the plants, which are brittle, in half. μηδείς (talk) 21:03, 14 October 2012 (UTC)[reply]
I have heard of putting chicken wire on the ground to prevent squirrels digging up flower bulbs. Maybe something similar would work.75.41.109.190 (talk) 22:51, 14 October 2012 (UTC)[reply]
The plants I have in mind are kept in pots, moved outdoors in March, and indoors at the end of October, and replanted yearly. I have already moved the last one indoors this afternoon, on the assumption that the two extra weeks I might get outside will not be worth the work, cost, and risk.μηδείς (talk) 23:21, 14 October 2012 (UTC)[reply]
Sounds sensible. If you want something attractive, you definitely don't want chicken wire. StuRat (talk) 08:14, 15 October 2012 (UTC)[reply]
You bury the chicken wire just under the surface of the soil in the pot. You don't see it, but it makes it difficult for a squirrel to dig.209.131.76.183 (talk) 13:46, 15 October 2012 (UTC)[reply]

I just wanted to mention how successful and popular StuRat's suggestion to hang the squirrels from trees using a long wire has been. Three relatives, a loved one, and two local librarians (they buy huge poinsettias yearly, and I have convinced them to take them home and reflower them rather than throw them out) have all broken out laughing when I relayed his advice. μηδείς (talk) 21:48, 15 October 2012 (UTC)[reply]

Glad I could help, if only by making them wet their pants while laughing. :-) StuRat (talk) 02:45, 17 October 2012 (UTC) [reply]
I've had my own run-ins with squirrels. One came down my chimney, made himself at home, found a loaf of bread, chewed it up and formed it into little balls. He apparently planned to take them back up the chimney and store them for winter, before I came home and, with great difficulty, ejected him from my house. On the plus side, I seemed to scare him enough not to try that again. StuRat (talk) 02:49, 17 October 2012 (UTC) [reply]

Honors thesis project

I am currently researching coastal armoring for my Honors Thesis. I am looking to expand this Honors Thesis into a senior project that I can present at the Northeastern Geological Society of America conference. I still am in the process of creating a thesis statement for the project. Does anyone have any suggestions, specifically about what questions or issues about coastal armoring (particularly in Connecticut are the best to explore? I intend on doing fieldwork of my own for this project. Thanks.--99.146.124.35 (talk) 18:26, 14 October 2012 (UTC)anonymous[reply]

It would have been helpful to explain what coastal armoring is. Apparently it's methods used to prevent shore erosion. I would think a cost/benefit analysis of various methods would be a good thing to study. That is, which methods are more economical than simply abandoning the shoreline and moving inland ? Another option is to look at the effects of global climate change on shore protection efforts. I suspect that methods formerly used are no longer effective when faced with rising sea levels and increased storm intensity. StuRat (talk) 19:08, 14 October 2012 (UTC)[reply]
Coastal management may be of interest. hydnjo (talk) 19:20, 14 October 2012 (UTC)[reply]
How about: which zones are more prone to erosion than others; which are zones are most important to protect, from an environmental perspective, and from an infrastructure perspective; is it economical to have your cake and eat it, if not what compromising schemes are available that balances both?Plasmic Physics (talk) 23:22, 14 October 2012 (UTC)[reply]
  • My first thought is, how does greenhouse effect interact with coastal armoring, especially if the collapse of major ice shelves occurs more quickly than imagined? There is a paper [19] which makes a number of recommendations for "federal/regional government actions", e.g. "Congress should amend the Coastal Zone Management Act (CZMA) to require relevant state agencies to consider sea-level rise in coastal management plans in order to qualify for federal funding assistance; prohibit federal subsidization of infrastructure development and coastal armoring in areas subject to sea-level rise; and encourage public and private land acquisition of coastal habitats and upland buffers." I'd think your project might explore whether the list of actions recommended by the organizations that produced this report for Florida are things which Connecticut should endorse or oppose. (I have absolutely no idea at all myself) Wnt (talk) 00:27, 15 October 2012 (UTC)[reply]
  • I think that you are going to have to narrow your topic, otherwise you will have too much to read and write about! Graeme Bartlett (talk) 10:38, 15 October 2012 (UTC)[reply]

How about the economical difference between manual replenishment with bulldozers and temporary pumping rigs versus a permanent replenishment system similar to water or sewer systems? Usually there are certain places that always get the extra sand and places that always lose it.165.212.189.187 (talk) 13:04, 15 October 2012 (UTC)[reply]

StuRat, read Armor (hydrology). 2001:18E8:2:1020:749C:5B76:1D8E:3D22 (talk) 16:45, 15 October 2012 (UTC)[reply]

Magnetic propulsion

I've read that magnetic motor is impossible due to magnetic equilibrium, but in this video for example a computer cooler seems to spin very well due to repulsion between magnetic poles (that is, one pole interacts with several opposite ones, constantly pushing them). I've browsed the internet further, but still don't get why the magnetic repulsion cannot be used as a source of free mechanical energy (especially considering strong magnets).--176.241.247.17 (talk) 21:41, 14 October 2012 (UTC)[reply]

Magnetic propulsion requires electromagnets (not just permanent magnets) and they are powered by electricity. Therefore, it's not "free".StuRat (talk) 21:56, 14 October 2012 (UTC)[reply]
Magnets apply a force, but force is not energy. This is a distinction which can be very counterintuitive and there are many people who have spent a LOT of time pursuing the futile dream of extracting free energy from a magnet. Our Free energy device article has asection which discusses magnets. Vespine (talk) 23:52, 14 October 2012 (UTC)[reply]
Indeed, many people have wasted years of life trying to get free engergy with magnets. It's also been a way to scam others. Take Steorn, for example, who took millions of dollars of investors money on the promise of "unlimited free energy". Needless to say they produced nothing of value, and failed year after year to prove any of their claims.217.158.236.14 (talk) 11:02, 18 October 2012 (UTC)[reply]

Flooded skyscrapers

In the movie A.I. Artificial Intelligence, Manhattan is flooded due to global climate change. However, the top floors of at least one skyscraper continue to be occupied. My quetions:

1) I assume that flooding existing skyscrapers would quickly cause the supports to rust out and fail. Is this correct ? How quickly ?

2) Is there a cost effective way to retrofit the existing supports to make them survive ? Some type of waterproofing ?

3) Could new supports be added to the existing building in a cost-effective manner ? StuRat (talk) 22:17, 14 October 2012 (UTC)[reply]

It was an artistically appealing image. Corrosion to failure should take decades, at the most.[20]. Why would anyone with the resources to do so retrofit those buildings a portrayed in that movie? μηδείς (talk) 23:16, 14 October 2012 (UTC)[reply]
Well, if they could be retrofitted at a lower cost than creating new buildings, that would be a reason. Or, in the case of significant historic buildings, they might be preserved even if this costs more than rebuilding. StuRat (talk) 23:19, 14 October 2012 (UTC)[reply]
You might want to link to an article on contemporary retrofitting. I don't think you'll find much on the history of retrofitting in the distant future. μηδείς (talk) 23:59, 14 October 2012 (UTC)[reply]
It's a sci-fi scenario, so there are a lot of possibilities. For example, you could produce some sort of nanobots that home in on sources of dissolving iron and chemically modify any exposed surfaces to passivate them. Or at least you might have more convenient drones for working underwater that you can use to rustproof the exposed materials more easily. There's also the simpler possibility that stronger fireproofing materials that have been or will be developed in the wake of the September 11th attacks turn out to be so tenacious that they block water corrosion as well as a bonus.Wnt (talk) 00:31, 15 October 2012 (UTC)[reply]
It's a sci-fi scenario, so it's asking for speculation, opinion, and debate, and it should be hatted. We have no relevant references. μηδείς (talk) 00:33, 15 October 2012 (UTC)[reply]
Just because you don't have a source, that doesn't mean there isn't one. Note that numerous sci-fi scenarios, like communications satellites, later became reality. StuRat (talk) 00:41, 15 October 2012 (UTC)[reply]
^ "Just because you haven't a source..." hydnjo (talk) 03:27, 15 October 2012 (UTC)[reply]
Fixed, but not in your manner, as that's entirely too British English for me. :-) StuRat (talk) 05:33, 15 October 2012 (UTC) [reply]
So, in the future, buildings that are already standing today, will become corrosion resistant because...it's the future? Let's just please provide some refs if we're going to continue this one. μηδείς (talk) 00:45, 15 October 2012 (UTC)[reply]
A lot depends on how much flooding you have in mind, and probably also the rate of flooding. I don't recall the scenes in the movie that you are referencing, so I'm not sure what scale of inundation you have in mind. Defending against 2 to 6 feet of sea level rise (typical projections for this century if global warming continues), probably could be accomplished with sea walls, pumps, and other basic barriers. However, complete melting of Greenland (20 ft of sea level equivalent) and Antarctica (200 ft equivalent), would seem very hard to fight against. On the other hand, humans do build oil platforms over 250 ft that are fixed to the sea floor (as opposed to the floating ones used in very deep water), so the idea of skyscrapers that are grounded hundreds of feet under water probably isn't impossible, though I wouldn't want to speculate on what issues would be involved in such a conversion. Dragons flight (talk) 01:29, 15 October 2012 (UTC)[reply]
It was hundreds of feet. The Statue of Liberty was submerged to the base of the torch (but the World Trade Center was alive and well).StuRat (talk) 05:29, 15 October 2012 (UTC)[reply]

Bioconcentration

Is it feasible to use cultivars to bioconcentrate soil-polutants, such as heavy metals, which can then be removed through harvesting the plants?Plasmic Physics (talk) 23:08, 14 October 2012 (UTC)[reply]

Certainly! Phytoremediation. Vespine (talk) 23:14, 14 October 2012 (UTC)[reply]
Possible, but doesn't seem very feasible to me. For one, you end up with a large volume of plant matter containing a small percentage of heavy metals. What do you do with that ? StuRat (talk) 23:15, 14 October 2012 (UTC)[reply]
Burn it. This is rather basic. μηδείς (talk) 23:17, 14 October 2012 (UTC)[reply]
Then you risk releasing some of those heavy metals into the air. Mercury vapor, for example, is bad stuff. There's also the risk that insects or animals will eat the plants, at which point the heavy metals enter the food chain, which is exactly what we want to avoid. StuRat(talk) 23:20, 14 October 2012 (UTC)[reply]
More specifically Phytoextraction process describes pretty much exactly what the OP is asking about. Burning it might be the best thing to do in some specific isolated cases, but I doubt it's a good idea as a rule of thumb. Vespine (talk) 23:21, 14 October 2012 (UTC)[reply]
Gasification? Plasmic Physics (talk) 23:25, 14 October 2012 (UTC)[reply]
Is there a plant that concentrates mercury? I have heard of those that do arsenic and copper. Surely this would be done industrially, and not in back yard barbecues. μηδείς (talk) 23:26, 14 October 2012 (UTC)[reply]
No terrestrial plants naturally concentrate mercury to a significant degree. There are currently efforts at multiple laboratories to engineer such plants with genes from bacteria that metabolize organic mercury (the bacteria use this to salvage hydrocarbons). In theory, this would cause inorganic mercury to concentrate in the plants, and it seems to work in the lab: [21].Someguy1221 (talk) 23:56, 14 October 2012 (UTC)[reply]
Maybe no terrestial plants, but Plankton certainly concentrate mercury, a casual search has not revealed to me if that also specifically applies to phytoplankton, or any other members which might be considered "plant", (if there are any). Vespine (talk) 23:58, 14 October 2012 (UTC)[reply]
What if the the deposits the polutant in nodule-like growths, which renders it inert? The plant would ideally act as a molecular pump, not simply as a spunge. Plasmic Physics (talk) 01:34, 15 October 2012 (UTC)[reply]
GMO is not excluded. Plasmic Physics (talk) 01:36, 15 October 2012 (UTC)[reply]
Nice idea. If we could do that, we could also finally mine seawater, bΝy having those plants collect all the substances we want and sequester them in those nodules for us to collect later. Get those genetic engineers to work ! StuRat (talk) 05:25, 15 October 2012 (UTC)[reply]
Sorry, I'm not good with sarcasm; is that genuine? Plasmic Physics (talk) 05:33, 15 October 2012 (UTC)[reply]
(My sarcasm is always genuine.) But no, this is not sarcasm. It may all be possible, some day. StuRat (talk) 05:40, 15 October 2012 (UTC)[reply]
Well, that's bitter sweet - it may be a million dollar idea, but I'm not capable of undertaking such endevour. Plasmic Physics (talk) 06:16, 15 October 2012 (UTC)[reply]
Maybe not sponges (not sure) but sea quirts have vanadocytes which concentrate the metal to an extraordinary degree. Of course, that's only a few cells, and so it isn't really economically feasible as encountered, yet... it inspires the imagination. Wnt (talk) 14:01, 15 October 2012 (UTC)[reply]
So someday we may be able to use them to sponge off the ocean ? :-) StuRat (talk) 16:17, 15 October 2012 (UTC) [reply]
In my comment, I was not refering to Porifera. I as refering to the final destination of the polutants - by "as a sponge", I was refering to the polutant remaining inside the cells, poisoning them. The second scheme, which I suggested, is for the polutant to be transported and sequestered from the cells at a common destination, leaving the plant unaffected. A sponge can only absorb a limited amount of water before it becomes fully saturated, a pump can transport as water as long as it is supplied with energy. Plasmic Physics (talk) 06:48, 16 October 2012 (UTC)[reply]
StuRat, you may be surprised to find that we do mine seawater: Magnesium#Occurrence. Buddy431 (talk) 16:49, 15 October 2012 (UTC)[reply]
Great, now I want to mine gold and platinum. :-) StuRat (talk) 06:50, 17 October 2012 (UTC)[reply]
Research has shown that dust sourced from road surfaces, is a rich source of valuable metals such as rhodium and palladium. Plasmic Physics (talk) 10:29, 17 October 2012 (UTC)[reply]
To help any less technical people who are reading this - as that article says, magnesium is present in about 1/8 the quantity of ordinary sodium (in salt), which we also "mine" from the sea. Wnt (talk) 17:07, 17 October 2012 (UTC)[reply]

October 15

Malaria in Africa

The earlier thread Malaria in Europe mentioned that malaria had been essentially eliminated in Europe and North America during the twentieth century. I hadn't realized that there had been that much success in eliminating malaria in regions where it had gotten established. My question is, how different is malaria in Africa from malaria in other places? Do differences such as climate and different mosquito populations make it much harder to fight in Africa? Or is it more a matter that the economic resources aren't available to fight malaria in Africa? If I had a huge sum of money (say $100 billion) and complete freedom to go anywhere in Africa and employ the same aggressive techniques that worked previously in the US and Europe, would that be enough? Could you wipe out the malaria bacterium, or is there some reason why techniques that worked in the past couldn't work in Africa? Dragons flight (talk) 02:40, 15 October 2012 (UTC)[reply]

There is an on-going global effort to eradicate malaria. Have a look at this and these papers. Zoonoses (talk) 04:53, 15 October 2012 (UTC)[reply]
You might run into people who think you are trying to poison them when you spray their land with DDT. StuRat (talk) 05:22, 15 October 2012 (UTC)[reply]
Malaria spreads when a mosquito bites an infected person, after which the plasmodium gametocytes differentiate, then reproduce. Then the mosquito has to bite someone else and rub off enough protists to get them sick. If malaria patients can afford to stay at a hospital until they're treated, where there are presumably fewer mosquitoes, that hampers the plasmodium lifecycle much more than if they had to continue working outdoors. --140.180.242.9 (talk) 05:59, 15 October 2012 (UTC)[reply]
The most effective methods used in the past were A) completely draining all standing water from a region and B) carpet bombing an area with DDT. Both methods are unpopular nowadays, in addition to the longstanding lack of any financial incentive to ridding Africa of malaria. I also suspect that the environment in sub-Saharan Africa is far less tenable to both methods, due to the sheer amount of warm, wet land. Someguy1221 (talk) 09:04, 15 October 2012 (UTC)[reply]
$100 billion wouldn't begin to do it. Africa contains on the order of a million square miles of prime mosquito habitat. Note that malaria is also endemic throughout much of South America, which contains more millions of square miles of prime mosquito habitat. Looie496 (talk) 16:08, 15 October 2012 (UTC)[reply]
That works out to $100,000 per square mile. Is that really not enough to spray a square mile with DDT, especially if you employ natives at the local pay rate (and contribute to the local economy in the process) ? StuRat (talk) 06:47, 17 October 2012 (UTC)[reply]
WHO estimates US $6B annually would do the job (over a number of years). [22]. Zoonoses (talk) 05:36, 18 October 2012 (UTC)[reply]

Baumgartner vs Kittinger

Trying to put Felix Baumgartner's jump into perspective, without all the media hype. Some questions:

  1. Why did it take more than 50 years from the previous height record of Joseph Kittinger? Is it so much more difficult to reach heigher altitudes or is it rather a lack of scientific value (of taking people up there) combined with the cost involved? Were there any attempts at breaking the record of Kittinger before?
  2. What is the main achievement or difficulty to overcome in the Baumgartner dive?

bamse (talk) 08:09, 15 October 2012 (UTC)[reply]

1) It's basically just a stunt, as there's no real need for people to be able to bail out at such heights. StuRat (talk) 08:12, 15 October 2012 (UTC)[reply]
2) Take your pick: high speeds, cold temps at the start, high temps when first hitting thicker atmosphere, lack of air pressure, etc. StuRat (talk) 08:18, 15 October 2012 (UTC)[reply]
2) Yes, but Kittinger had the same, so my question is, whether there is any additional factor to take into account for Baumgartner which did not matter for Kittinger. Adding 3) Did Baumgartner and Kittinger use essentially the same tools and techniques, i.e. is the only difference that Baumgartner had more modern tools/better materials? bamse (talk) 08:27, 15 October 2012 (UTC)[reply]
Adding 4) It is often claimed that it is the first supersonic flight of a human without "vehicle". On the other hand, Baumgartner was naturally not on his own but in a "suit". How much does this suit differ from a supersonic "vehicle"? bamse (talk) 08:30, 15 October 2012 (UTC)[reply]
The most obvious difference is that the suit had no propulsion system - the speed attained was due to gravity's acceleration, making this qualitatively different from powered flight. As far as risks, I was under the impression that there were many - among the greatest being a flat spin resulting in circulatory derangement, loss of consciousness, death. Recovering from such a spin can be hard in a vehicle that has control surfaces and an engine, but Baumgartner just had his arms and legs. I'll try to provide a ref in a little while - I'm sure there are suitable ones. -- Scray (talk) 08:46, 15 October 2012 (UTC)[reply]
Here's one: Reuters Factbox: Greatest risks of high-altitude skydiving, which says that the greatest risk was suit breach or accidental (early) parachute deployment. Another, from National Geographic discusses 5 principal risks as low pressure, cold, wind/weather, flat spin, and the hard-to-anticipate risks of breaking the sound barrier without a craft (as discussed above). -- Scray (talk) 09:07, 15 October 2012 (UTC)[reply]
Thank you. The National Geographic site you linked answered all my questions. bamse (talk) 18:29, 15 October 2012 (UTC)[reply]
In February 1966 Nick Piantanida did in fact go higher than Kittinger, and also higher than the officially planned height of Baumgartner but not the actual height. However, Piantanida didn't make the planned jump and didn't return with the balloon either but in the gondola with a large parachute. Our article has a poor description of the February flight. Click "Long Description" at [23] to see what happened. PrimeHunter (talk) 00:06, 16 October 2012 (UTC)[reply]

Perpetual motion

When did scientists first understand that perpetual motion machines are impossible? I can't find the answer in our articles. Duoduoduo (talk) 14:41, 15 October 2012 (UTC)[reply]

Like most scientific knowledge, this was a gradual development, as scientists began to formalize their understanding of experimental results. I'd start by reading our article on thermodynamics, especially the history section. For example, it's doubtless that Isaac Newton had a pretty good idea that friction slowed objects in real systems; and by the time Carnot wrote about formal engine thermodynamics, he stated the laws of thermodynamics outright. If you run farther back into history, Aristotleian physics was simply wrong, and Aristotle asserted the existence of perpetual motion machines without evidence. So, somewhere along the way, we figured out that perpetual motion didn't exist, and gradually made our explanations more clear and consistent with all other physical law. Nimur (talk) 15:04, 15 October 2012 (UTC)[reply]
  • The impossibility of perpetual motion is a direct consequence of conservation of energy. That principle was first clearly formulated by Hermann von Helmholtz sometime around 1850. So there couldn't have been a really compelling rejection of perpetual motion before that time. However, as our History of perpetual motion machines article says, "In 1775, the Royal Academy of Sciences in Paris made the statement that the Academy "will no longer accept or deal with proposals concerning perpetual motion." The reasoning was that perpetual motion is impossible to achieve and that the search for it is time consuming and very expensive." Looie496 (talk) 16:03, 15 October 2012 (UTC)[reply]
We do have the article History of perpetual motion machines. Leonardo da Vinci (15th century) designed some devices that he hoped would be perpetual motion machines, but upon studying them, decided that it probably wasn't possible. Buddy431 (talk) 17:05, 15 October 2012 (UTC)[reply]
I, personally, do not think that a perpetual motion machine of the second kind, which does not violate energy conservation, is impossible though, because I've figured out how to build one. With it, the empirical second thermodynamic law is no longer an obstacle (its scope being severely limited, of course, by virtue of the existence of my process, which I claim) and we can therefore get rid of fuels and put a stop to global warming. -Modocc (talk) 17:32, 15 October 2012 (UTC)[reply]
I saw your plans for it scribbled in the margin of some book you were reading:) DMacks (talk) 17:52, 15 October 2012 (UTC)[reply]
I fear that making my work public is thorny though, for a provisional patent and a subsequent public disclosure is not always adequate with regards to the all important step of reduction to practice. I have to be careful to include all the necessary details (unlike Fermat's scribbles). -Modocc (talk) 18:13, 15 October 2012 (UTC)[reply]
Unfortunately the US Patent Office has a policy of requiring a working model before granting a patent for a perpetual motion machine. Looie496 (talk) 18:32, 15 October 2012 (UTC)[reply]
Patent models have not been required in US patent applications since 1880. Someguy1221 (talk) 23:42, 15 October 2012 (UTC)[reply]
Ah, I found what you were referring to. The US patent and trademark office specifically requires perpetual motion machine applications to be accompanied by a working model, due to their tendency to be full of shit. Perpetual motion#Patents. Patents are still occasionally granted to such "inventions", however. Why, I don't know. Someguy1221 (talk) 23:47, 15 October 2012 (UTC)[reply]
Probably because patents are granted for all sorts of complete nonsense. A patent means nothing until it is tested in court. And if the patent holder can convince someone to buy the rights, well and good, regardless of whether the patent is truely valid or not. It is not the patent office's problem if you have wasted your money getting a patent that makes no sense or has no commercial value. The US Patent Office's decision on obvious perpetual motion devices probably arose from a desire to avoid dealing with complete wankers or perhaps a public service desire to prevent retards wasting their money. An example of a patent that is just as much nonsense thermodynamically as perpertual motion is US Patent 4945866, Altered Piston Timing, Bertin R Chabot, published 08/07/1990, describing an internal combustion engine in which the center line of each cylinder is offset from the rotational axis of the crankshaft in the direction of rotation. This patent is an amazing collection of nonsense claims showing a total lack of understanding of basic thermodynamics, but the idea gets represented under different titles and wording every 10 to 15 years or so. An example of a patent that does not violate basic thermodynamic principles but has no commercial merit whatsoever, a fact that is obvious at a glance to any engineer (it's full of flat surfaces and parts that can't be cooled, for a start), is the Sarich Engine. An example of a patent that is clearly invalid due to prior art is the Dolby noise reduction patent - it never the less was a huge commercial success due to excellent marketing. Ratbone121.215.28.169 (talk) 00:40, 16 October 2012 (UTC)[reply]
That is correct, obviously. -Modocc (talk) 01:20, 16 October 2012 (UTC)[reply]
A working model is needed for acceptance by the examiner. But, that only establishes its patent-ability to the patent office's satisfaction, subject to its appeals process, but the date of the examination (as with all appeals) will differ from the date of reduction to practice which will be earlier, and as you have pointed out, needs not be the date a working model is completed. -Modocc (talk) 00:21, 16 October 2012 (UTC)[reply]
That shouldn't be a problem, for I'm fairly certain that provisional patent application(s) can establish a reduction of practice, as long as it specifies how to build the working model that is submitted at a later date. My fear is that I may leave out some minor detail with my application(s) that prevents this. Similarly, should I consider simply putting it into the public domain anywhere with a full public disclosure, but this disclosure happens to be slightly incomplete or needs slight correction, it may not be considered a reduction to practice, and whoever recognizes such mistake(s) and fixes these with some mere paperwork may claim the invention! This is therefore a difficult dilemma for me to resolve. Also, further complicating matters, my skills (my eyesight is deteriorating and I'm getting too impatient to stay focused on my work) at actually building the machine are very limited too, but it is simple enough that I am in the process of building it (the parts and tools I've needed for it has cost me less than eighty bucks, but lately its been languishing on my desktop, as its only partially assembled). I've got to do more... -Modocc (talk) 18:47, 15 October 2012 (UTC)[reply]

Our speed in day vs night

Since we travel at 1674k/h around the earth and the earth travels at 108000k/h around the sun, during the day we travel 3300k/h slower than at night relative to the sun. Does this have any noticeable effect on anything?165.212.189.187 (talk) 14:58, 15 October 2012 (UTC)[reply]

Yes, that's what causes sun's contribution to the tides. Dauto (talk) 15:13, 15 October 2012 (UTC)[reply]
(ec) I first thought that ths might have an impact to aiming large telescopes; aligning the telescope and compensating for Earth's rotation would require some added complexity in addition to a standard equatorial mount. But I'd never heard of any telescope using such a compensation, so I went looking through the webpage of the largest telescope that came to mind: Control Systems for the 30-meter telescope. Everything is adaptive and closed-loop control now; atmospheric compensations, laser guide stars; and so on. These telescopes use feedback control to drive the scope the exact optical center of their imaging target. It's not clear from this high-level overview whether those mechanisms and algorithms explicitly include a correction factor for sidereal / solar rotation deviations; but it seems like that would be either unnecessary or superfluous given the already elaborate system. My suspicion is, if you work the math, the angular error to a distant object is negligible, whether you account for Earth's revolution around the sun or not; but you should be able to work out the math and see exactly how tiny that non-zero deviation actually is, and you can even calculate the error as it changes during a single day or night.
In other contexts, satellite communications require complicated doppler shift compensations, and the rotation of the planet should be included in this compensation. I once knew a guy who bounced a signal off of Mars, and the return echo sounded like a descending whistle. I can't recall which time-varying doppler shift was responsible for that; but I sort of seem to recall that it was the rotation of the transmitter on the surface Mars that caused it. I'll try to dig up the actual publication, which should explain the details with a bit more scientific rigor. Nimur (talk) 15:20, 15 October 2012 (UTC)[reply]
"My suspicion is, if you work the math, the angular error to a distant object is negligible"
Indeed. Let's work the math. Close stars like alpha Centauri show a parallax of about 0.7 arc seconds due to the earth orbiting the sun (which is ~1000 light-seconds or 300 million km measuring the orbit end to end). These 0.7 arc seconds are too small to be seen without a telescope but provide a good method of calculating distances between stars. The parallax Earth adds with its rotation is much smaller. You're looking at ~12700km back and forth, which would introduce a daily parallax of ~0.00003 arc seconds for alpha Centauri. This is a noticeable effect when looking at the Moon (it seems to wobble from side to side) but utterly insignificant when looking at the stars.
You could look at it this way: Average stars are about 100 times the size of planet Earth. Now you'd need a telescope capable of resolving a star into 100x100 pixel to see it wobble one pixel back and forth per day. Today's telescopes are not capable of resolving any star except exceptionally huge ones (huge telescopes looking at huge stars, that is). And... don't you think that could be easily remedied by, say, taking one photo per minute and then match and combine these photos to eliminate the motion blur? My digital camera has that kind of image stabilizer, so it's near-trivial to an observatory-grade telescope.
However, all Doppler experiments, including exoplanet detection, must compensate for the velocity itself. Earth goes around the Sun at about 29km/s, and Jupiter is about half that (~13km/s). Since Jupiter is 1/1000 the mass of the Sun, the Sun moves at 1/1000 the speed of Jupiter, i.e. ~13m/s. (If I got the math right, that is. I did it mentally) Compare that to the rotation of Earth, which is 40000km in slightly less than 86400 seconds, or ~480m/s at the equator. If orbit and/or rotation were not accounted, for even Jupiter-sized exoplanets in large orbits would be utterly undetectable. - ¡Ouch! (hurt me / more pain) 07:05, 18 October 2012 (UTC)[reply]
Oops. Don't take the "480" too seriously, it's about 460. I was doing the number-crunching mentally. Yeah, using my own blood for ink. ;) - ¡Ouch! (hurt me / more pain) 13:01, 19 October 2012 (UTC) [reply]

Single original chloroplast?

(1) On p. 64 of the October 2012 National Geographic magazine it says:

Chloroplasts...evolved about 1.6 billion years ago when one cell, incapable of using the sun's energy, engulfed another cell -- a cyanobacterium -- that could. That cyanobacterium became the ancestor of every living chloroplast.

And our article chloroplast says:

All chloroplasts are thought to derive directly or indirectly from a single endosymbiotic event (in the Archaeplastida), except for Paulinella chromatophora, which has recently acquired a photosynthetic cyanobacterial endosymbiont which is not closely related to chloroplasts of other eukaryotes.[4]

And the abstract of citation [4], which is all I can access, says:

There is accumulating evidence to support a single primary origin of plastids from a cyanobacterium (with one intriguing possible exception in the little-studied amoeba Paulinella)
(a) In layperson's terms, what is the nature of the evidence for a single originating event?
(b) Isn't this single-ancestor assertion highly implausible on the face of it? It seems that if conditions were amenable for this event and its propagation at that time, it would have occurred numerous times with the descendants surviving.

(2) Same questions for the original cell or the original self-replicating molecule. (I know I've seen it asserted quite a few times that all cells or self-replicating molecules have a common original ancestor, though I can't recall a particular source that says it.) Duoduoduo (talk) 16:36, 15 October 2012 (UTC)[reply]

For question (1), the crucial thing is that chloroplasts, like mitochondria, contain their own DNA, separate from the DNA in the cell's nucleus. It is possible to compare chloroplast DNA for various plants and other photosynthesizing organisms to get an estimate of how long ago their evolutionary ancestry diverged.
For question (2), the main reason is that there are a number of complex metabolic mechanisms that are shared by all existing organisms. Our article on the evolutionary history of life gives references for that statement, if you are interested in more information. Looie496 (talk) 16:48, 15 October 2012 (UTC)[reply]
On point (1b), saying that all organisms of a given line (or all living organism) came from single events doesn't mean that the events were themselves the only instance of such an event before or since, just that the other lines from the other "events" have gone extinct. Just to use another example, Mitochondrial Eve wasn't the only Homo sapiens woman at the time she lived. She's just the one whose matrilineal line didn't eventually get subsumed by another (or die out) over the hundreds of thousands of years since then. --Mr.98 (talk) 16:50, 15 October 2012 (UTC)[reply]
Good answers above. I'll just note that (at a glance), the article does seem to contain a good overview/review of the related work that has constitutes this "accumulating evidence" of single-origin chloroplasts. The content may or may not be easily accessible, depending on your background. Drop me a line if you'd like me to send you the article. SemanticMantis (talk) 16:55, 15 October 2012 (UTC)[reply]
"Careful" science too often involves deliberate semantic gaming to be vague. "A single event" or "a cyanobacterium" might not refer to a single chloroplast. It is possible that a wide population of billions of symbiotic cyanobacteria was being taken up from the outside world by cells for a long time, then the cells gradually picked up genes from the cyanobacteria, which led to individual bacteria separately losing them under selective pressure, etc. The divergence of the individuals within a single population of a single species is far too small to show up against the larger mutational noise after all this time; so far as I can imagine (which is not proof...) there is no way to prove it was or was not a single individual chloroplast that produced the rest. Wnt (talk) 19:00, 15 October 2012 (UTC)[reply]
I don't think that's actually what they're saying. The two clades simply correspond to the most widely branched groups of plants. Wnt (talk) 23:56, 15 October 2012 (UTC)[reply]
This text

The paraphyly of Guillardia and Odontella, with respect to the two red algae, also suggests independent acquisition of secondary chloroplasts in the heterokont and cryptophyte, in contrast to the hypothesis of a single secondary endosymbiotic event among the chromophytes (Cavalier-Smith 2000). Although a single origin of the chloroplasts in this group is supported in some analyses (De Las Rivas, Lozano, and Ortiz 2002; Yoon et al. 2002), the topology of these four taxa in our tree is identical to that based on a recent, traditional analysis of protein-coding genes in the genomes (Martin et al. 2002). Analysis of small subunit ribosomal DNA in the chloroplasts from a wide variety of rhodophytes and chromophytes also indicates that chloroplasts of the latter group have independent origin

and this diagram indicating separate rhodophyte and chlorophyte lineages seem to indicate at least two separate events. μηδείς (talk) 00:45, 16 October 2012 (UTC)[reply]
Well, secondary chloroplasts are surrounded by three or four membranes and are the remnants of a larger endosymbiont. I think all the varieties mentioned are in reference to the secondary events. [24] emphasizes that all, in the beginning, came from one event. Wnt (talk) 01:26, 16 October 2012 (UTC)[reply]
Yes, I am familiar with that concept from reading I did in the 90's, although I don't fully know how to interpret what I am reading here. It seems strange that secondary endosymbiosis would have more origins than primary. I would think the bottom line would be, is there a phylogeny that shows one type of chloroplast is more closely related to one type of bacteria, while a second type of chloroplast is more closely related to a second type of bacteria than it is to the first type of chloroplast. μηδείς (talk) 01:41, 16 October 2012 (UTC)[reply]
Here is something interesting: a few months ago, in vivo research has indicated that an endosymbiotic event is close to completion, the event would result in a new generation of plants which capable of nitrogen fixation. The event is creating a new plastid termed a nitroplast.
One cell (a prymnesiophyte), incapable of nitrogen fixation, is engulfing another cell (Candidatus Atelocyanobacterium thalassa) -- a cyanobacterium that can. That cyanobacterium will become the ancestor of every future nitroplast. Plasmic Physics (talk) 07:17, 17 October 2012 (UTC)[reply]
They stress that the particular species of prymnesiophyte may not be the only host cell involved in this event, it is just the only one encountered thus far. Plasmic Physics (talk) 07:23, 17 October 2012 (UTC)[reply]
Cool! Let's decorate this with some sources: [25] [26] [27] Wnt (talk) 06:33, 18 October 2012 (UTC)[reply]

Most Recent Common Ancestor - Pre-Columbus

I was perusing the article Most_recent_common_ancestor, and I was quite surprised to see it proposed that "With the advent of mathematical models and computer simulations, researchers now find that the MRCA of all humans lived remarkably recently, between 2,000 and 4,000 years ago." Of course, this is followed by a [citation needed] tag.

Assuming that this is the accepted time frame, the article states that this is attributable to the introduction of the genetic material of the European colonizers around the world. My question is, have pre-contact remains in the Americas or Australia been tested to determine how far back they would have shared a common ancestor with, say, the Spanish conquistadors?

I'm not expecting people to come to this conclusion, but I'm amusing myself about how it would certainly turn things upside down if it were discovered that Columbus and the poor Native American individuals he abducted and took to Spain as specimens were removed from the same ancestor by only 1500-3500 years. Falconusp t c 21:58, 15 October 2012 (UTC)[reply]

I've been wondering about this recently too. Does the claim that the MRCA lived at most 4,000 years ago imply that every now-living person (including Australian aboriginals, and members of 'uncontacted' South American and Indian Ocean tribes) is descended from someone who lived before even the earliest 'modern' transatlantic contact, but long after the rise in sea levels and corresponding vanishment of ice bridges that placed Alaska out of reach of Asia, and Australia out of reach of Southeast Asia? AlexTiefling (talk) 22:10, 15 October 2012 (UTC)[reply]
I'm hunting for a paper that even makes that claim, but it's clear to me it's not a universal opinion. This paper from earlier this year, looking at particular populations amongst Asians and Native Americans is sticking to the more common claim of ~15,000 years. They do speculate that populations often described as "isolated" for however-many years may have had occasional interlopers entering their community from later migrations, reducing the age of the MRCA. Anyway, still looking for a source for the 2-4kya claim, which does seem kind of ridiculous. It's going to be heavily dependent on what model they used to produce that number. Someguy1221 (talk) 22:40, 15 October 2012 (UTC)[reply]
PMID 15457259 may be what you are looking for. The basic idea is that everybody throughout the whole world has at least some recent European ancestry. There is certainly no claim that the Indians who Columbus met had recent common ancestry with him -- the claim is all of their descendants have picked up some European blood in the meantime. I have edited the MRCA article a bit to make it clearer that the statement in question was referring to the material in the following two paragraphs. Looie496 (talk) 23:32, 15 October 2012 (UTC)[reply]
I have removed the claim, it's absolute nonsense. The source is one author, duplicated by citing the same work in a preliminary form that explicitly says not for citation. One computer modeling program cannot override the clear evidence that there are pure aboriginal populations in the Americas and Australia with pedigrees at least 15,000 years separated from the old world, let alone Europe. μηδείς (talk) 23:50, 15 October 2012 (UTC)[reply]
The proper reference would be the one I cited above (PMID 15457259), which covers the same material and was published in Nature. Looie496 (talk) 00:28, 16 October 2012 (UTC)[reply]
I don't have a huge problem with citing the study if it is attributed to the authors and if the implication, that there are no surviving pure Amazonian or Australian aborigines is made explicit. But the study is basically a report about a computer model, not about reality, (i.e., if you write a program with such premises you get such results,) and I don't see it meriting any weight. μηδείς (talk) 00:59, 16 October 2012 (UTC)[reply]
I haven't read the article, but from the abstract, I agree with Medeis. It is only a "what if" computer experiment, useful only if you are trying to establish the validity of other similar computer models. --Lgriot (talk) 07:39, 16 October 2012 (UTC)[reply]
Well, I think that the modelling study (which I also have not read) could be indicative of the facts - I'll see if I can get a friend with PubMed access to get me a copy. But yes - we should avoid repeating its conclusions as though they are the facts on the ground. AlexTiefling (talk) 09:31, 16 October 2012 (UTC)[reply]
SomeGuy - isn't that paper talking about MRCAs specifically for mitogenetic elements? If I've understood correctly, that means we'd be looking at specifically matrilineal common ancestors, and those could easily be an order of magnitude more ancient than the general MRCA. But maybe I've misunderstood (again). AlexTiefling (talk) 09:41, 16 October 2012 (UTC)[reply]
You're absolutely correct, and it's something I should have mentioned, and yeah, it doesn't really touch on the general MRCA (it just provides an upper bound). It's worth noting that MRC-patrilineal-A and MRC-matrilineal-A are much more active research subjects than the basic MRCA as they are easier to handle (virtually no recombination for Y-chromosomes or mito-DNA). Someguy1221 (talk) 03:45, 17 October 2012 (UTC)[reply]

Alright, thanks all! I thought it was really interesting, but then if it really came out that the genetics were that mixed up, it would have made for more than a brief note on a Wiki Article. Falconusp t c 03:29, 17 October 2012 (UTC)[reply]

Well, let's suppose we start by imagining a possible MRCA living in the southern Urals around the time of the Trojan War (1100BCE). Let's suppose a moderate reproductive rate of 3 children per parent per generation surviving to reproduce. The 100-year rule implies that by the time of Julius Caesar, this person would have 30 generations' worth of descendants. If all of them were completely exogamous, that would be 3^30 = 200 trillion descendants. This is obviously absurd - but it's certainly credible for everyone in Eurasia, Africa and the surrounding islands at that date to be descended from our putative MRCA - mostly many times over. The same logic applies to the succeeding 1000 years - any one person alive at the time of Caesar could be an ancestor to the whole Old World by the time of Erik the Red. If just one of Erik's son Leif the Lucky's crew successfully reproduced with a 'skraeling' inhabitant of the New World just once, and that child survived to reproduce at the previously assumed rate, that child might have around 5 million descendants in the New World at the time of Columbus's arrival.
Meanwhile, on the other side of the world, the Macassans have been visiting northwestern Australia in their canoes for centuries. As they're based on the Asian mainland and adjacent islands, they're almost certainly all descended from our Uralic MRCA by now. Again, it only takes one of them to reproduce with one Australian Aboriginal person for a similar logic to apply to that entire population at the time of Europeans' first direct contact with them. So it's not unbelievable for the MRCA of all humanity to have been within 3000 years of Columbus at the time he set sail. I wouldn't want to guess, though, at the date of the universal MRCA at the time of Erik the Red. That might have been as long as 10,000 years previously. AlexTiefling (talk) 11:11, 17 October 2012 (UTC)[reply]

October 16

2 questions: Human powered flight and looking for the name of a theory

While watching the video in this article from NPR about the Sikorsky Prize for human powered flight, it made me wonder. Why would they build a system where the pilot has to use both his arms and legs? Wouldn't he be able to put more power into his legs if he could use his hands to brace himself? It seems to me that it would also be more fluid of a motion.

Also, it brings to mind a theory/principle/something that I read about here years ago. Basically it said that if you have a group of people pulling on a rope, the combined pull will have less force than if you were to add up their individual pulling forces when pulling one person at a time. Can anyone name what I'm thinking of? Our article used tug of war as an example but I can't find the term that I'm looking for when I look through what links to that article. Dismas|(talk) 00:15, 16 October 2012 (UTC)[reply]

Doing a bit more reading, University of Maryland Gamera II Human Powered Helicopter says that "Up to 20 percent additional power for the 60 second runtime is achieved using this more complex method rather than pedal power alone." But it doesn't say why or cite a reference. Dismas|(talk) 00:35, 16 October 2012 (UTC)[reply]
The reason is simple: By using both arms and legs, more muscles are brought into use, so greater power. For sustained effort, finite lung capacity comes into effect - evolution has provided us with lungs that are insufficient for maximum output from all muscles simultaneously, but for 60 seconds or so each muscle can run on oxygen held in the blood, and CO2 levels don't rise too much. Wickwack124.182.43.72 (talk) 01:13, 16 October 2012 (UTC)[reply]
Erm... doesn't swimming use both arms and legs at the same time? Alansplodge (talk) 12:53, 16 October 2012 (UTC)[reply]
I must admit I'm not a good swimmer, however I can swim. It seems to me that one's arms get to be used at something like full power, but while the legs contribute, they don't operate at maximum effort. Something that does use both, plus other muscles as well, at high effort is rowing in a racing skiff. I've never done that, but such races seem to be very short in duration. Wickwack 124.182.187.189 (talk) 14:56, 16 October 2012 (UTC)[reply]
The contribution of different muscle groups depends on style and speed. For freestyle, about 80% of the force comes from the arms. For sprints, legs are important, but for long-distance swimming, you mostly only use the legs to keep the body in a good horizontal position. Those big leg muscles eat up oxygen like there is no tomorrow... --Stephan Schulz (talk) 21:14, 16 October 2012 (UTC)[reply]

Law of diminishing return / utility?GeeBIGS (talk) 00:57, 16 October 2012 (UTC)[reply]

I found the answer to my second question!! I trolled through my contributions a bit and it is... The Ringelmann effect! Dismas|(talk) 07:05, 17 October 2012 (UTC)[reply]
Awesome. I added that to the article I wrote on diseconomies of scale. StuRat (talk) 07:22, 17 October 2012 (UTC)[reply]

Combined energy source vehicle ?

Extending the above question, wouldn't a plane that was neutrally buoyant (mostly filled with helium, or, if you dare, hydrogen), and covered with solar cells, in addition to being human powered, do better than one which is human powered alone ? (I realize this is likely against the rules of the competition.) StuRat (talk) 06:04, 16 October 2012 (UTC)[reply]

A lot better, though the solar cells are not going to help much. You're going to need some way of getting down though. See Larry Walters.--Shantavira|feed me 07:22, 16 October 2012 (UTC)[reply]
StuRat's idea has been proposed before, but it's not a good idea. It depends on what you define as better. Sure you can use helium or hydrogen to make a craft neutrally boyant in air, but the required volume means it will have a big surface area and will want to go where the wind takes it, so you'll need a bigger power input to make it go where you want. This is why you see conventional winged airplanes carrying freight, but not airships. The power to payload ratio of winged aircraft overtook airships many decades ago. Wickwack 124.182.187.189 (talk) 10:31, 16 October 2012 (UTC)[reply]
Good idea or not; it is in serious development as we speak; Not only do we have Lockheed Martin’s HALE-D airship takes to the air but also Solar Ship: The hybrid airship with a low-carbon twist. Alansplodge (talk) 12:51, 16 October 2012 (UTC)[reply]
When they go into commercial production that might mean something. Meanwhile I classify them with nuclear aircraft engines (http://en.wikipedia.org/wiki/Nuclear_aircraft), Reagan's StarWars technology, and Rover gas turbine engines for cars - all of which got very serious funding, and all of which were totally dumb things technically to do, and consequently were scrapped. Wickwack 120.145.19.16 (talk) 01:35, 17 October 2012 (UTC)[reply]

I realize this idea isn't practical as far as a commercial way to transport people or cargo. However, it might have recreational possibilities. That is, it might be a reasonable alternative to things like hang gliders, hot air balloons, and ultralights. And, just like those, you would be restricted to days with low wind, and, due to the use of solar panels, you'd also need bright sunlight. StuRat (talk) 02:17, 17 October 2012 (UTC)[reply]

Fair enough. But what we should do here is ask does combining gas buoyancy and power give us a synergistic advantage (or the best of two worlds) or does it just combine disadvantages? There's no doubt that hot air ballooning has considerable followers - I've tried it myself and its great fun, especially when you turn that great noisy burner off. Forget hydrogen - one bit of static discharge and your gone. Would helium catch on? Well, its pretty darn expensive - accepable for niche commercial purposes but too expensive for recreation. Now, does adding propulsion power to helium buoyancy give an advantage? Well yes it does, but once you have implemented propulsion power, regardless of the power source (gasoline engine, solar cells or whatever) you are better off with wings, as explained. That is, combine human power with solar power not in a buoyant lift craft but in a winged craft. But that's for chaps with olympic-standard fitness. What might have some synergistic recreational merit is a winged craft with solar cells covering the wings for cruising power, plus a small gasoline engine for take-off power. It should be possible as technology advances to get the weight of the solar cells and electric motor lower than the fuel weight for cruising range, and stay aloft for a long time that way. Wickwack 124.182.144.228 (talk) 02:46, 17 October 2012 (UTC)[reply]
A gasoline engine would add weight, fumes, and noise. They could jump off of hills like hang-gliders do, to get started. And yes, my idea was hydrogen or helium filled inflatable wings with flexible solar panels on top (hydrogen is only flammable when mixed with air). I believe an unmanned vehicle was made like this, and stayed aloft for many days. But, my version wouldn't have batteries for night storage of energy and would add a pilot who could hopefully control the flight direction with a leg/solar-powered, turnable fan. The price of helium is a concern, especially if it was all just vented to deflate the aircraft for storage. Is there a way it could be sucked out and recompressed back into the tanks ? StuRat (talk) 06:39, 17 October 2012 (UTC)[reply]
I'm sure there is, though it might cost some significant energy. The problem is that if you have neutral buoyancy at some altitude, to descend you must either vent some helium (the usual way back in the 1930's), or compress it on board, which has been proposed before. The power to weight ratio of constant-speed gasoline engines is excellent, however maybe you should calculate how much cranking a human would need to do to control altitude by manual onboard gas compression. Don't be too concerned about the noise of an IC engine - you should hear the noise of a hot air ballon gas burner in full cry just above your head. My usual statement for this type of discussion on Ref Desk applies: Should a hand compressed helium lift wing work out, with or without photo-voltaic boosting, don't bother patenting it, folks, as by posting all these response StuRat & I have established prior art. Wickwack 124.178.143.72 (talk) 08:38, 17 October 2012 (UTC)[reply]
Perhaps make the vehicle slightly heavier than air, so that modest pedaling or sunlight on the solar panels provides sufficient thrust to provide the extra lift needed, via the airfoil. StuRat (talk) 08:52, 17 October 2012 (UTC)[reply]
I wondered if you'd come back with that. You'd need a lot more than modest pedalling if you got caught in a quite modest thermal. Wickwack 124.178.143.72 (talk) 10:19, 17 October 2012 (UTC)[reply]
Just like a rip tide, you don't fight against a rising thermal, you get out of it first, then land. StuRat (talk) 01:34, 18 October 2012 (UTC)[reply]
Re hydrogen only flammable when mised with air (or oxygen) - yes, that's obviously true. Trouble is, it is the leakiest stuff imaginable, that tiny molecule can permeate such a range of things. Its pretty hard to guarantee it doesn't get mixed with air somewhere, it is combustible down to very low concentrations, and ignitable by the tiniest static discharge spark. Best avoided. Wickwack 124.178.143.72 (talk) 08:38, 17 October 2012 (UTC)[reply]
Well, there is some risk, but I do feel the risk has been overstated as the result of the Hindenburg disaster (where the source of ignition was likely a flammable surface treatment). After all, most aircraft contain flammable fuels, it's just a matter of properly containing them to prevent explosions. Isn't there any flexible material which is impermeable to hydrogen ? StuRat (talk) 08:52, 17 October 2012 (UTC)[reply]
As the WP article covers quite well, many and various are the theories why the Hindenburg burned, and we cannot really know, but the official and most likely cause is a static electricity discharge. While aircraft fuel (gasoline or kerosene) is regarded as highly inflamable, there is a very considerable difference between these fuels and hydrogen. For a start, aircraft fuels being hydrocarbon liquids, they must first be vaporised and pyrolised (ie the molecules split up) before combustion can occur. Once vaporised they can all be ignited by a spark, but the required spark energy is very high in comparison. To ignite gasoline you need a clearly visible spark at least about 1 mm from any (cold) surface; for hydrogen you need only a spark too way too weak to be seen, and it can be right up against a surface. It's true that the extensive media coverage at the time including the live voice coverage turned the public off airships. But that doesn't change the fact that hydrogen filled airships, certainly of that design, were dangerous. Wickwack 124.178.143.72 (talk) 10:57, 17 October 2012 (UTC)[reply]
And yet, none of the 119 other Zeppelins suffered mysterious mid-flight explosions -- even shooting them with incendiary or explosive anti-aircraft rounds wasn't a reliable way of setting them on fire. --Carnildo (talk) 01:22, 18 October 2012 (UTC)[reply]
Neither did the Hindenburg suffer any mid-flight problem. The hazards come when coming down to land, and when tethered. Wickwack 121.221.89.122 (talk) 02:42, 18 October 2012 (UTC)[reply]

Will canned food become hard to taste before the rust level becomes harmful?

Though if you eat it without the liquid the majority of the rust taste disappears. Hey I'm poor, I'm gonna eat it if up to 60% of the flavor is rust. Otherwise I throw away a dollar. Of course it also matters how often you eat from rusty cans and how many a lifetime. Why do most of them not taste like rust but a few taste bad? And I can't imagine how crappy the unlucky ones taste like by the time they're the "best by" age, which are like 12 times older than the ones in the supermarket). 96.246.59.161 (talk) 05:03, 16 October 2012 (UTC)[reply]

Whatever you're smoking, can I have some of it? Looie496 (talk) 05:40, 16 October 2012 (UTC)[reply]
I assume that English is a second language for you, and you meant to ask "Will I be able to taste that canned food is off before it becomes harmful, due to age ?". Most of the time, yes, but occasionally it might develop something nasty with no bad taste. I also don't think it's typically rust that you taste, it's more the coatings on the inside of the can and the oxidation of the contents. StuRat (talk) 05:53, 16 October 2012 (UTC)[reply]
I suspect this is getting a bit close to medical advice. FWIW from experience as a student it always seems that acid tinned stuff (grapefruit segments, tomatos) does get rusty. But although people have eaten rusty stuff no one is going to advise you its ok. The main ingredient of rust is Iron(III) oxide which is insoluble and not going to do much and if the manufacturers of the tin had you in mind there might not be seriously bad stuff. --BozMo talk 05:55, 16 October 2012 (UTC)[reply]
It's worth remembering that as StuRat hinted at, the rust may not be your biggest concern with the tins. It's whatever has happened to the food due to whatever is growing on it thanks to the rusting/compromised tins. Nil Einne (talk) 11:49, 16 October 2012 (UTC)[reply]
Cans are always lined, and they don't rust on the inside. If they are damaged enough to expose the interior to rust, that's a very dangerous situation -- but it hardly ever happens. Most cans nowadays won't rust at all, unless exposed to something corrosive. The OP here is just making stuff up. Looie496 (talk) 14:57, 16 October 2012 (UTC)[reply]
Sure but if you can actually taste the rust as alleged by the OP or are going to be eating rust because of a rusting can as discussed by BozMo, then the dangers of the rust should not be high on your list of concerns. Nil Einne (talk) 15:02, 16 October 2012 (UTC)[reply]
What's in the coating that you taste it? Tin? It doesn't stop the 1.5 mm wide seam from rusting. Clearly if the can is damaged or has enough rust to eat it should be thrown out but I never saw one like that. 96.246.59.161 (talk) 23:22, 16 October 2012 (UTC)[reply]
Not sure what the metallic taste is. I've taken a bite from inside a can before, only to spit it right out due to a metallic taste, then check the expiration date and wonder if the can came with bell bottom pants and a rainbow colored disco wig. :-) StuRat (talk) 02:10, 17 October 2012 (UTC)[reply]
Beer in aluminium cans develops a metallic taste eventually, so rust isn't the cause in that case. Alansplodge (talk) 19:08, 17 October 2012 (UTC)[reply]

Why don't protons repel each other ?

As we know, protons are positively charged particles and it is also known that like charges repel each other. But protons occur in nucleus and they don't seem to repel each other. Why it is so ? What make protons and neutrons stick to each other ? Sunny Singh (DAV) (talk) 10:41, 16 October 2012 (UTC)[reply]

See nuclear force and strong interaction. Sean.hoyland - talk 10:51, 16 October 2012 (UTC)[reply]
(ec) Protons do repel each other by electromagnetic forces, but at the same time, the Nuclear forces hold the protons and neutrons together. These nuclear forces work only at very small distances. Even then, neutrons are necessary to stabilize the nucleus because these contribute to the binding by means of nuclear forces, but they are electrically neutral do they do not repel each other or protons. - Lindert (talk) 10:57, 16 October 2012 (UTC)[reply]
Actually neutrons stabilize the nucleus by reducing its Fermi energy - that's why stable nuclei have about as many protons as they have neutrons. Only for large nuclei does the electric energy becomes somewhat important - that's why heavier nuclei have an excess of neutrons (compared to their number of protons). Dauto (talk) 14:02, 16 October 2012 (UTC)[reply]
You can imagine the nucleus (especially heavy nuclei) as a balance of contradictory forces. When it gets out of balance, little bits can shoot out (radioactive decay) or, in extreme cases, the whole thing can rupture into two pieces (nuclear fission). Understanding that protons do repel each other, except at very small distances, also helps make sense of why nuclear fusion is so hard — you have to get the protons close enough to each other for the nuclear force to win out over the electromagnetic force (the Coulomb barrier — illustrated here). --Mr.98 (talk) 14:54, 16 October 2012 (UTC)[reply]

If someone give further explanation of my first question, I'll thankful to him.
We know that protons and neutrons are made up of up and down quarks. I want to know which particles constitute electrons ? Sunny Singh (DAV) (talk) 02:00, 17 October 2012 (UTC)[reply]

I believe that the Standard Model posits that electrons (and other leptons) are elementary particles, not composed of other particles. --Trovatore (talk) 02:04, 17 October 2012 (UTC)[reply]
What particle contains three down quarks? Plasmic Physics (talk) 02:12, 17 October 2012 (UTC)[reply]
The Δ- baryon. Dragons flight (talk) 02:21, 17 October 2012 (UTC)[reply]
See also the list of baryons, for the composition, names, and symbols for all of the baryons. TenOfAllTrades(talk) 02:26, 17 October 2012 (UTC)[reply]
What is unclear to you about the examples already given to you to the first question? The basic answer (the nuclear force) is discussed above, along with some more complicated refinements (e.g. Fermi energy). Protons do repel one another because of the electromagnetic force, but at short distances the nuclear force is dominant, which causes nucleons to attract to one another. The linked-to articles above go into nearly endless technical details. Just let us know what is unclear and someone will happily elaborate or translate into laymen's terms. --Mr.98 (talk) 02:45, 17 October 2012 (UTC)[reply]

Yes, you are right Mr.98, linked articles provided enormous information of my first question. But, what about my second question. Sunny Singh (DAV) (talk) 11:16, 17 October 2012 (UTC)[reply]

Re-read Sean Hoyland's answer. --Jayron32 11:57, 17 October 2012 (UTC)[reply]

Thermodynamics of melting salted ice

Hello,

Considering a bath of ice at -10°C to which NaCl salt is added on surface, hence forming a layer of eutectic ice NaCl•H2O (melting at -22°C instead of 0°C), the Gibbs free energy of the melting eutectic ice in a 25°C thermostated-environment will roughly have the following form:

Experimentally, the use of a thermometer will show that the salted ice goes to -22°C when melting, hence showing that the first term of the equation (integral from -10°C to -22°C) describes the first physically occuring response of the system when heated to 25°C. This integral being the only negative term of the equation (entropy aside), my question is: which energy is transferred to the environment during the -10°C → -22°C transition? Would it be a heat transfer? Thanks, 188.194.48.183 (talk) 11:10, 16 October 2012 (UTC)[reply]

The temperature drop that occurs when the salt is added to the water comes from the net endothermic solvation process. The breaking of the Na-Cl ionic bonds is more endothermic than the formation of the solvent-ion bonds is, so it's just an endothermic heat of solution. That's why the temperature of the solution drops: the energy is being used to cause the NaCl to dissolve. --Jayron32 21:44, 16 October 2012 (UTC)[reply]
That makes a lot more sense put that way. Thank you! 188.194.48.183 (talk) 20:25, 17 October 2012 (UTC)[reply]

Likely heat situation of the ~26-mile BASE jumper yesterday

Regarding the man who went up in a helium balloon and jumped from what I'm hearing is about 26 miles altitude and reaching about 834 mph, I saw in a brief shot on the news that he was wearing a sort of space suit that seemed to be made of flexible fabric material. I wonder, given the composition of the atmosphere at the point at which he was going 834 mph, what was the likely temperature (ballpark figure) due to friction? 20.137.2.50 (talk) 13:37, 16 October 2012 (UTC)[reply]

That's a good question. The suit Felix Baumgartner was wearing was a heated pressure suit, intended to keep him at a comfortable air pressure, and nice and warm, in the near vacuum of such high altitudes. That 834 mph was possible because, at that time, he was falling through so little air that there was little friction acting on him. The terminal velocity at low altitude is about 120 mph. So Baumgartner will have slowed from 800 to maybe 100 mph before he deployed his parachute. That speed will all have been lost due to friction (most in the last minute or so). Handwavingly assuming Baumgartner, suit, chute, et al weighed in at 200kg, I figure his kinetic energy was 12.7 MJ at the fastest point, down to about 0.2 MJ when he was at ~100 mph - meaning he'd have lost 12.5 MJ of kinetic energy. Not knowing much about the aerodynamics of falling spacesuits, I don't know how much of that went to eating up the air, and how much to heating up the suit. -- Finlay McWalterTalk 14:08, 16 October 2012 (UTC)[reply]
Hopefully most of that energy heated up the air! I estimated a little bit differently, assuming that all the gravitational potential energy needs to convert to heat (well, most of it, anyway); so I got about 35 MJ using the standard "PE = mgh" formula. To order of magnitude, it's the same as Finlay's estimate. And, assuming a human is mostly water, with a specific heat capacity of 4.18 J/g·K, that's enough energy to raise a 90 kg human by almost 100 degrees centigrade! That's probably fatal! Of course, the rate that heat is conducted and radiated to the surrounding air is very rapid; and a human in a space suit is not well-approximated by an equal mass of water; but to first order, we have to assume that the majority of that thermal energy is lost to air and/or ablation of the suit - hopefully in an engineered fashion. Nimur (talk) 15:56, 16 October 2012 (UTC)[reply]
[28] says that a failure of the heater in his face plate was not an issue in the capsule but would be on jumping. This confirms your impression that the rapid cooling effect from the passing -60 degree air is overwhelming the heat from reentry. Reentry from space involves much more lost potential energy and much less cooling by contact... in any case, I think your estimate and answer puts us in the lead! :) Wnt (talk) 17:06, 16 October 2012 (UTC)[reply]
I am sure there was some heating effect but I doubt it was a large one. 800 mph is fast, but is still very much less than the speeds of spacecraft re-entering the atmosphere from low Earth orbit - about 16,000 mph - or the speeds at which meteorites enter the upper atmosphere - at least 25,000 mph. Gandalf61 (talk) 14:55, 16 October 2012 (UTC)[reply]


When he was going 800 mph, that was the "local" terminal velocity, which gradually became less as the atmosphere became more dense. So, when he was going 800 mph, the air resistance was equal to his weight, therefore the dissipated power at that time and later was his weight times his velocity and that was maximal right at that time when he reached maximal velocity. So, if he weighs 80 kg, the maximal dissipated power he experienced would be about 290 KW. This is way more than he could have sustained on the long run, but then this only lasted for a matter of seconds.

Another way to see that 800 mph is a problem, is to compare the velocity at which the air molecules hit you at that velocity with the how fast they hit you at rest and compute how high the temperature needs to be for this to be the same. You can roughly estimate this as follows. At rest at -40°C, <v^2> is about (455 m/s)^2 )(using the formula 3 k T/m), the component in the direction of motion is a third of this, so sqrt[<vz^2>] = 263 m/s. If we add to this the 800 mph, we get vz = 623 m/s, so relative to Baumgartner, sqrt[<v^2>] = (sqrt[623 m/s)^2 + 2 * (263 m/s )^2] = 725 m/s. This is the same as being at rest at a temperature of 317 °C. In reality it isn't the same, because the frequency of the collision is less at lower densities. Count Iblis (talk) 17:09, 16 October 2012 (UTC)[reply]

Additional complexity arises because these speeds are in the transsonic and supersonic regime; in other words, an air molecule traveling at 725 m/s at stratospheric densities is not in kinetic equilibrium. Each molecule is, in fact, in shock. I don't believe you can apply conventional gas temperature calculated from velocity according to the equipartition theorem, in this regime. Our article has information on attached shock, for molecules impacting the falling person; and detached shock, for the aerodynamic phenomenon that will inevitably form. Aerothermal heating in the transsonic and supersonic regime is widely studied. For the very enthusiastic thermodynamicist, NASA makes available an excellent free text, Facing the Heat Barrier: A History of Hypersonics - about the physics at much faster than super-sonic speeds. Aerodynamics of "hypersonic" flight are defined by the gas regime when thermal effects are more important than any other physical aerodynamic effect. This is usually around Mach 5 or higher. This is one reason that spacecraft reentry systems use reaction control systems, and not flaps or parachutes - because conventional aerodynamic control surfaces do not behave as expected at these speeds. This book mostly applies to hypersonic aircraft; but I also found the more conventional book, Entry heating and thermal protection, which seems to provide a lot more practical approach, covering material more applicable to jumping out of a balloon at 120,000 feet. Nimur (talk) 19:38, 16 October 2012 (UTC)[reply]
Let's expand this a little. 80 kg * 9.8 m/s^2 = 784 newtons (kg m/s^2) of weight. The negative work (energy released) done by this weight is force x distance = 784 kg joules (m^2/s^2) per meter fallen. We know he falls 0.447(m/s/(mi/hr) conversion factor)*(800 mile/hour) = 357 m/s, so the energy released is his weight 784 kg m/s^2 * his velocity (357 m/s) = 280000 kg m^2/s^3 = 280000 watts. If I didn't foul up, that's a lot of power - at least twice what an automobile engine produces, according to the article. But for each second of this power output he passes through 357 meters of the surrounding air, releasing the original 784 joules per meter. So the question is, how much of a temperature differential will it require to get the surrounding air to take up 784 joules as it rushes past? Hmmm.... Wnt (talk) 18:02, 16 October 2012 (UTC)[reply]

Too bad they couldn't have fit one more thing, a temperature sensor, into his chest pack, if only for my personal curiosity's sake :). An ounce of actual measurement is worth a ton of theory. But from the mission's own site regarding his pressure suit: "The suit is designed to provide protection from temperatures of +100°F to -90°F." I guess there's my ballpark figure from the people closest to the real and true answer 20.137.2.50 (talk) 20:50, 16 October 2012 (UTC)[reply]

So, from above, we have that he must be dumping 0.2 MW into the air at his peak velocity. Assuming an air density of 0.03 kg/m^3 at that height and an effective cross-section of say 0.8 m^2, that implies about 9 kg / s of perturbed air. With a specific heat of around 1 J / gram / K, that ballparks the heating of the air at around 25 Kelvin. So, it would seem that his problem is likely to be cooling due to low ambient temperatures rather than overheating due to friction, at least during the fastest part of his trip. Dragons flight (talk) 00:52, 17 October 2012 (UTC)[reply]

October 17

Chemistry to dissolve human poo?

What does human poo consist of? fats? protein? is it polar or non polar? and more importantly what is the chemistry that will dissolve it? just like alcohol dissolve oil etc. Electron9 (talk) 02:24, 17 October 2012 (UTC)[reply]

What is the chemistry? Don't you mean solvent? It's a complex mixture of both polar and non-polar compounds. Plasmic Physics (talk) 02:28, 17 October 2012 (UTC)[reply]
This topic is extensively covered in our article on wastewater treatment. Nimur (talk) 02:31, 17 October 2012 (UTC)[reply]
Seems to normally be low on fats, so will just dissolve in water with a bit of agitation. However, some people with non-functional gall bladders or using diet products like Orlistat do pass substantial portions of fat, so adding some detergent would cover those cases. StuRat (talk) 02:38, 17 October 2012 (UTC)[reply]
A significant amount of poo is cellulose, which isn't soluble in either detergent or water. --Jayron32 02:50, 17 October 2012 (UTC)[reply]
Ah yes, and there's always the corn kernels to deal with, along with seeds and perhaps grains of sand, which aren't going to dissolve in anything short of a powerful acid or base. StuRat (talk) 06:30, 17 October 2012 (UTC)[reply]
What's the best household- or easily obtainable chemical to dissolve cellulose ..? Electron9 (talk) 03:15, 19 October 2012 (UTC)[reply]

Mystery leaf 'streamers'

Maple leaf with 'streamers'

I took the photograph at right this weekend in central Ontario, Canada, about 100 km north of Toronto. It was on a small maple tree, on which a number of leaves had these dark, vertical 'streamers' (about 1 cm long) sticking out of their upper surfaces. Some leaves had just a few, others (like the one pictured) were heavily covered.

Can anyone tell me what these streamers are? KevinHadley (talk) 03:44, 17 October 2012 (UTC)[reply]

Maple spindle galls, caused by a mite. http://www.uoguelph.ca/pdc/Factsheets/Diseases/Maple_Galls.htm μηδείς (talk) 04:01, 17 October 2012 (UTC)[reply]
That's pretty neat, the mites are able to alter the leaf cells to grow a gall around them. Tiny genetic engineers at work. StuRat (talk) 06:27, 17 October 2012 (UTC)[reply]
Not genetic - it's mostly the result of hormonal effects. Roger (talk) 11:11, 17 October 2012 (UTC)[reply]
Neat. Thanks! KevinHadley (talk) 13:49, 17 October 2012 (UTC)[reply]
  • Wikipedia does have a short article on Galls, which may lead the reader to other information as well. --Jayron32 13:53, 17 October 2012 (UTC)[reply]

Why Lube oil consumption of Gas Engine increase after major overhauling.

"Why Lube oil consumption of Gas Engine increase after major overhauling — Preceding unsigned comment added by 182.182.122.148 (talk) 06:20, 17 October 2012 (UTC)[reply]

Is that a gasoline engine ? And did they mill down the cylinders ? StuRat (talk) 06:26, 17 October 2012 (UTC)[reply]
Gas engine or gasoline engine, taking it that it is in any case a spark ignition piston engine: In vintage engines, a temporary increase in oil consumption above normal tended to occur as carbon at the top of the piston formed a seal - the carbon got removed in overhaul and needs to build up again. However in modern engines, especially for bowl-in-piston/squish land piston/EE-type combustion chambers, a noticeable increase (as distinct from a modest increase) in oil consumption is a sign of errors in rebuilding. Posible errors are: cylinders not correctly honed, incorrect oversize piston and/or rings fitted wrt cylinder re-bore, piston rings incorrect or incorectly fitted, problems with valve guides. A possible problem is the oil presure control spring together with tight bearings causing higher oil pressure leading to over lubrication of valve gear. At one time, new engines and overhauled engines were initially filled with "running-in" low-viscosity oil, which also increases consumption, but this should not be done nowadays. Wickwack 124.178.143.72 (talk) 08:17, 17 October 2012 (UTC)[reply]

New exoplanet temperature

See [29]. That says the surface temperature is around 2200°F, due to it's proximity to the star. However, at that distance it's sure to be tidally locked to it's star (unless the planet is very young). If we also assume it to have no atmosphere or liquid covering it (having been blown off by the solar wind), wouldn't the dark side be far cooler ? StuRat (talk) 09:03, 17 October 2012 (UTC)[reply]

I don't see why not, without a thermal vector, there is no way for heat to be distributed except by good old fashioned conduction. I assume that you're aware of Mercury (planet). Plasmic Physics (talk) 10:35, 17 October 2012 (UTC)[reply]
StuRat, you beat me to it! I even wonder if outgassing from the magma side would lead to frozen atmosphere accumulating on the dark side, with the occasional cryovolcano of liquid water and room temperature air awaiting such Robinson Crusoes as our imaginations can devise. :) Wnt (talk) 16:54, 17 October 2012 (UTC)[reply]
(ec) This article about the newly discovered planet says near the bottom:
And whichever side of the planet faced the star would be broiling hot, with the other side icy cold.
This doesn't actually say that it's tidally locked, but it makes your point about the surface temperature not being high everywhere. Duoduoduo (talk) 17:00, 17 October 2012 (UTC)[reply]
Why do you mention Mercury in this context? Do you perhaps hold the common incorrect pre-1965 assumption that Mercury is tidally locked to the Sun, making its supposed dark side cold? Are you aware of Mercury (planet)? 88.112.36.91 (talk) 20:41, 17 October 2012 (UTC)[reply]
I was refering to the effect of a combination of a slow sidereal period, and a lack of atmosphere, to produce a large temperate gradient. Plasmic Physics (talk) 22:02, 17 October 2012 (UTC)[reply]
The article says that the dark side of Mercury averages just 110 K, with a "reaches 100 K at night" figure. I don't know if it would reach 77 K and allow nitrogen to condense out if it were locked 1:1 with its orbit. (or 90K for liquid oxygen, if it somehow came to exist; come to think of it, liquid methane at 110 K would be plausible, maybe, if it weren't constantly re-boiled and lost?) Clearly for any satisfying scenario with a reservoir of frozen/boilable atmosphere on Bb, a complete tidal lock would be preferred. But at 0.04 AU ... shouldn't it be? Wnt (talk) 22:13, 17 October 2012 (UTC)[reply]
I should correct myself: not slow sidereal period, but a ratio close to one, between sidereal rotational period and orbital period. A ration of 1:1 would indicate tidally locked. A ration of 1:1.5 for mercury is enough to produce a gradient. Plasmic Physics (talk) 01:31, 18 October 2012 (UTC)[reply]
Appearance
  • As an aside, I should mention that the artist's rendition from the article and as shown on our Main Page seems wrong to me. If it's hot enough to melt magma, shouldn't at least some of the planet's edge be visibly red? (see Incandescence, which starts at 525 C) Also, the graphic gives me the impression that from close by the planet, its star looks little bigger than the Moon; but it's only 20 times the distance from the Moon to Earth away from the planet, and nearly as big and half as bright as the Sun. Erm, to put that more simply, at 0.04 AU its star should look 25 times wider than the Sun, and I'm just not feeling that from the picture. Wnt (talk) 22:20, 17 October 2012 (UTC)[reply]
The surface may not glow bright enough to be visible with the star in the field of view. Also, if it is tidally locked with no atmosphere, then only the part illuminated by the star would be hot (that was the point of Stu's question), and that glow would be drowned out by the starlight. As for the sizes, that's going to depend on how far away the "camera" is from the planet and the star. The star has about half the apparent diameter of the planet. Since the star is actually about 100 times bigger (assuming the planet is about the size of Earth), that suggests the distance from the the camera to the Sun is about 200 times the distance from the camera to the planet. That puts it about 30,000 km away from the planet.
When viewed from the Earth, the Moon and Sun are both about 0.5 degrees in diameter. Alpha Centauri B from 0.04 AU would 11 degrees (just under 25 times wider). There is no way to know what the angular diameter is from an image like that, though - you would need to know what the total field of view was. --Tango (talk) 18:55, 18 October 2012 (UTC)[reply]

perverted justice

I've taken the liberty of moving this question to Wikipedia:Reference desk/Humanities#Perverted Justice (moved from Science Refdesk). Trust me, this is a good thing - you'll get better answers. Wnt (talk) 16:49, 17 October 2012 (UTC)[reply]

does a nurse need to expel air from a prefilled flu vaccine

when administering a prefilled flu vaccine does the nurse need to expel any air from the syringe — Preceding unsigned comment added by 95.146.101.33 (talk) 10:54, 17 October 2012 (UTC) |}[reply]

Two answers: [1] in general, no, because prefilled syringes generally don't contain air. [2] But if a nurse noted air, in general, he would expel it out of an abundance of caution rather than because it presented any real danger. As a rough estimate, it would take at least 20 cc of air injected directly into the bloodstream to cause an air embolism. A flu vaccination is about 0.5 cc: even if the whole syringe were air, it wouldn't be fatal. And it's also not going into the bloodstream, but intradermally or intramuscularly. See The Straight Dope for more discussion of how much air can be fatal. - Nunh-huh 16:19, 17 October 2012 (UTC)[reply]
Air injected into a muscle or under the skin is not particularly dangerous -- it might hurt like hell, though. Looie496 (talk) 16:27, 17 October 2012 (UTC)[reply]
If you Google this question, you actually can find multiple bulletin boards of nurses discussing a wide variety of practices. It seems that some vaccines recommend you do, and some recommend that you don't, and most don't say anything at all. I'm not sure there's any real consensus on it. But this is just a Google survey; I have no direct knowledge of this topic. --Mr.98 (talk) 22:44, 17 October 2012 (UTC)[reply]
I was recently self-injecting Clexane subcutaneously daily for a few weeks, every prefilled syringe had a small amount of air in it, and the instructions specifically stated "Do Not Remove the air before injection". I asked a few nurses and doctors about it, and their conclusion was that "amateurs" would probably expel half the drug trying to get the air out, and that small amount of air sub-cutaneously would do no harm at all.124.191.177.92 (talk) 07:42, 18 October 2012 (UTC)[reply]
Depending on the design of the syringe, there is often a dead volume (why is that red?) that is not expelled when the plunger is pushed all the way in. If the syringe is prefilled "to contain" the stated volume of liquid, the dead volume represents an amount of the stated volume that does not get delivered. If there is a small amount of air also, one can push out all the liquid because the air can be what remains in the dead volume. Calibrating "to deliver", one would include extra volume of liquid to compensate for what gets retained in the dead volume. I have no idea if this is how/why medical folks do what they do, but it's what I've seen done in other syringe-transfer work. DMacks (talk) 02:22, 19 October 2012 (UTC)[reply]

Human "fuel efficiency"

We tend to think of biking and walking as "green" alternatives to driving, but it seems to be this may not be true in some cases. Walking and biking don't use fossil fuels, but inasmuch as they require greater physical exertion, they could lead to more eating—human fuel, if you will. Since the production, transportation, and preparation of that food is almost never carbon neutral, could a person be making a more environmentally responsible decision by driving in some cases? If we were to graph it, I could see car efficiency increasing on longer trips. And perhaps hybrid or electrical cars could shift the graph a bit. --BDD (talk) 20:58, 17 October 2012 (UTC)[reply]

Keep in mind that fossil fuels are non-renewable (at least not for millions of years) and also release net carbon dioxide which was previously safely sequestered underground. Also, you assume that biking and walking cause you to eat more. Hopefully you lose some excess weight, instead. Then there's the improvement on your health and the inefficiency in caring for unhealthy people (with diabetes and such) to consider. There's also the concept that using a car more wears it out, while using the body more (within limits) actually makes it last longer. But, I suppose, if you have a future solar/battery powered car which is fully charged (so wasting any additional sunlight), then using it might not be bad alternative. StuRat (talk) 21:05, 17 October 2012 (UTC)[reply]
If you do the measurement of human efficiency, you have to keep in mind that when you walk/bike to work, you are moving a far smaller mass than if you were driving a car. It's about energy per person-mile. I believe my car weighs over ten times what I do, so I only have to be 10% as efficient as the car to break even. Someguy1221 (talk) 22:25, 17 October 2012 (UTC)[reply]


BDD is on the ball and has asked a very good question. In Mark's Standard Handbook for Mechanical Engineers, 11th ED, EA Avallone et al eds, 2007 McGraw-Hill, it says on page 9-5 that the [maximum useable, continous over a work day] power output of a typical healthy adult human is about 400 W, and when compared to the amount of food required to sustain this, the thermodynamic efficiency is about 25% - about the same as a typical 4-stroke gasoline engine under optimal speed and load conditions, but nowhere near as good as a modern turbocharged diesel engine (~40 to 45%). It also says that the maximum human mechanical power output (typical fit adult male) is about 1500 W sustainable for only 0.6 seconds. It should be noted that under typical driving conditions, a car gasoline engine is not operating at optimal load, so the real efficincy will be less than 25%. Incidentally, when I was about 16, I took up intensive weight training at the local YMCA. My appetite increased dramatically, and I've had trouble controlling my weight ever since. The fuel efficiency of fit humans is surpisingly good - old books give data for utilizing animals such as horses to power mills, water pumps, etc, the calorific value of the feed required makes them woeful compared to the IC engine, probably because typical work animals (horses, oxen) all eat food full of cellulose, which has calorific value but is difficult to digest. Ratbone 124.178.52.68 (talk) 00:11, 18 October 2012 (UTC)[reply]
However, supposing that vigorous cycling makes you eat an extra loaf of bread a day, that apparently equals around one kilogram of extra carbon dioxide.[30]. According to this table you get 328 grammes of CO2 when you drive one mile in a petrol (gasolene) car or 327 in a diesel one. So you get about 3 miles in your car instead of a loaf of bread. (PS I'm not a scientist, so there may be a serious flaw in my thesis). Alansplodge (talk) 18:20, 18 October 2012 (UTC)[reply]
But the carbon in that loaf of bread came from the atmosphere through photosynthesis, so you breathing it back into the atmosphere just gets you back where you started - it's just the carbon cycle. The problem with fossil fuels is that the carbon was sequestered underground for millions of years and would have stayed there had we not burned it. That introduces more carbon into the carbon cycle, which is the problem. --Tango (talk) 19:05, 18 October 2012 (UTC)[reply]
You're using the wrong metric, though - we're interested in energy per mile, not energy efficiency. The amount of energy required to travel a mile is going to be very different for a car than a person walking. A car travelling at 60mph has to use a lot more energy to overcome air resistance than a person walking at 3mph. As SomeGuy mentioned, a car is a lot heavier than a person. I don't know the numbers, but I expect cars use far more energy than people to travel the same distance, so even if the car was 100% efficient it would still need more fuel. --Tango (talk) 19:05, 18 October 2012 (UTC)[reply]
How much fossil CO2 was released in the course of converting 8g of wheat seed in a farm supply store into a loaf of bread in the OP's kitchen 500km from the farm. That's the real question that needs to be answered. Roger (talk) 19:52, 18 October 2012 (UTC)[reply]
To answer your's and Tango's points, the first site that I linked to is talking about carbon footprint rather than carbon capture. An 800g loaf couldn't possibly store 1000g of CO2. So the 1kg of CO2 that I quoted includes all the fossil fuel used in the production, processing and retailing of the loaf. Alansplodge (talk) 21:24, 18 October 2012 (UTC)[reply]
I think in the spirit of the OP's question it would be best to consider that the car is one of those biodiesel-fueled vehicles that runs on used cooking oil. Renewable fuels are great, but there are still limits on how much carbon we can pull out of the atmosphere with the available arable land. Wnt (talk) 20:13, 18 October 2012 (UTC)[reply]

October 18

Redstone

Meh, could someone clarify what is redstone here (concerning the Union Soldiers and Sailors Monument)? Looks like some sort of rock, but there is virtually no info on it in the internet and everything is flooded with Minecraft stuff (and no article in Wiki either). Brandmeistertalk 10:57, 18 October 2012 (UTC)[reply]

My guess is that only a description is provided by that term rather than an identification. I would guess that what is known is only the color—red. It is some unidentified red stone. Bus stop (talk) 11:06, 18 October 2012 (UTC)[reply]
The OED simply has "stone of a red colour" for this word, but apparently it can also mean red ochre, however that seems unlikely in this context.--Shantavira|feed me 12:29, 18 October 2012 (UTC)[reply]
The term is non-specific, and can refer to any type of stone. Could be sandstone, shale, mudstone or even granite or limestone. The only thing you can say for sure is that it sorta kinda looks reddish. As nothing of the sort is visible in any pictures of the monument, it probably refers to the cheaper stone that makes up the bulk of the pedestal, and is sheathed with the more attractive and expensive granite and marble. Dominus Vobisdu (talk) 12:56, 18 October 2012 (UTC)[reply]
This color photo of the monument shows that the pedestal has a reddish tint to it.    → Michael J    13:36, 18 October 2012 (UTC)[reply]
Most likely because it was taken at sundown or sunrise, with a very red sun. Other photos show that the granite is more gray than pink. Dominus Vobisdu (talk) 13:51, 18 October 2012 (UTC)[reply]
I expect you will find the most likely answer at http://www.anr.state.vt.us/dec/geo/redstone.htm, which describes a redstone quarry in Vermont. It states, "the rock is a reddish- purple rock called Monkton Quartzite". Looie496 (talk) 15:45, 18 October 2012 (UTC)[reply]
The linked article states; "New Hampshire granite, redstone, and marble." There is a village called Redstone, New Hampshire where there are quarries yielding "coarse constructional stones, all biotite or biotite-hornblende, but varying in colour, pinkish ("red") and dark-yellow greenish-grey ("green") varieties being found remarkably near each other at Redstone, on the east side of the Saco valley. The finer varieties take a high polish and are used for monuments..." Alansplodge (talk) 18:06, 18 October 2012 (UTC)[reply]
On a separate note, I think in the Infobox it should distinguish between the material the sculpture is made of and the materials the base is made of. Bus stop (talk) 21:33, 18 October 2012 (UTC)[reply]

Matching neurotransmitter and feeling

Can we quantify in ml and name neurotransmitters responsible for each feeling? For example, happiness -> 0.03 ml serotonine, and such. 83.52.248.109 (talk) 14:26, 18 October 2012 (UTC)[reply]

Unfortunately not. The closest we can come to matching neurotransmitters with feelings are probably "dopamine ↔ pleasure", "norepinephrine ↔ vigilance", and "oxytocin ↔ love", but those are all oversimplified. The role of serotonin has been incredibly difficult to understand, but "serotonin ↔ satisfaction" captures some aspects of it; happiness really doesn't.
Also, because neurotransmitters are targeted and channeled in a very precise way, quantifying them in terms of concentrations is not meaningful. It may make sense for hormones, which diffuse through the bloodstream, but not for neurotransmitters, which are released only at specific target sites. Looie496 (talk) 15:09, 18 October 2012 (UTC)[reply]
The effect of the neurotransmitter depends also on the sensitivity of the receptor and the reuptake of it. OsmanRF34 (talk) 15:40, 18 October 2012 (UTC)[reply]
  • You are making the reductive materialist mistake of equating a high-level emergent property with a physical substance. Happiness comes about because of the harmonious arrangement of the factors of your life. Are you healthy enough so you are not in a constant state of suffering, including chemical imbalances? Are you free from major worries about food, money, safety, shelter, the well-being of loved ones? Are your priorities in order, so that you are not frustrating your own goals? Do you have pursuits that bring you passion and pleasure? If your life, body and brain are well-organized and properly functioning you will be happy. This includes the proper levels and flux of hormones and neurotransmitters. But they work as lock and key mechanisms. There is nothing inherent in the shape of the neurotransmitter itself that causes happiness--it is whether it fits and stimulates the right keys, and the whole system works together properly to regulate the traffic of your mid that matters. You could genetically engineer the body to replace serotonin entirely with some sort of pseudoserotonin of another chemical makeup entirely. As long as you also replaced the serotonin receptors with pseudoserotonin receptors you'd be perfectly well off. There's nothing inherent in serotonin itself that conveys happiness. μηδείς (talk) 17:32, 19 October 2012 (UTC)[reply]

electrodynamics

Hello, sorry for being so ignorant, but what is the relation between classical electrodynamics and electronics? The thing is, electronics textbooks, at least the ones I've been reading (hobby-wise), they talk about analyzing circuits, the behavior of the different components and stuff, never about current density, displacement currents, electric flux and things, yet the stuff seems very important, the more so as Maxwell's equations play a role in relativity and stuff, or so I've read or heard. Are those things a higher-level abstraction, of sorts? Can Ohm's law be derived from Maxwell's equations? Asmrulz (talk) 18:01, 18 October 2012 (UTC)[reply]

Electrical circuit theory (Kirchhoff's laws and so on) is a special case of electrodynamics in which the electric fields are guided by conducting wires separated by insulators. You can derive these laws from the continuous equations of electrodynamics. Ohm's law and electronics are derived from a combination of electrodynamics and quantum theory, because quantum theory is needed to explain the behaviour of atomic lattices in resistors, charge carriers in semiconductors and electrons in vacuum tubes. --Heron (talk) 18:42, 18 October 2012 (UTC)[reply]

Coffee making soda go flat

I have on occasion mixed a teaspoon of instant coffee with a can of decaffinated cola. It makes the soda foam violently. I figured it was a mechanical interaction. But mixing coffee that's already been made seems also to make the coffee go flat. Any info on what's going on? μηδείς (talk) 19:28, 18 October 2012 (UTC)[reply]

Most probably Alzheimers. Ankh.Morpork 19:30, 18 October 2012 (UTC)[reply]
Nucleation, possibly with a side order of Diet Coke and Mentos? Certainly for the teaspoon of instant coffee, less sure for the already made. --Tagishsimon (talk) 19:36, 18 October 2012 (UTC)[reply]
I am not quite sure I get Pork's point, although I watched my mother's father die with Alzheimers--terrifying my sisters who were too young to understand his behavior before he became bedridden--not something to joke about. As for nucleation, yes, that's what I meant by "mechanical interaction". But I have noticed that even when the coffee is already well dissolved it still will flatten soda. μηδείς (talk) 20:34, 18 October 2012 (UTC)[reply]
Pork is just making a bad joke, I suppose. OsmanRF34 (talk) 00:21, 19 October 2012 (UTC)[reply]
Does coffee completely dissolve, or is there some element of particulate suspension? If so, that might account for nucleation. If not, then we seem to be left with weird chemistry. --Tagishsimon (talk) 20:50, 18 October 2012 (UTC)[reply]
I'm no scientist. However, I wonder whether this might hold a clue? It seems to suggest that caffeine itself has a surfactant effect in the soda/Mentos reaction, although not a hugely significant one compared to some of the other ingredients (see the slide comparing the performance of the normal and caffeine-free versions). A Google search brought up various relevant articles about coffee and bubbles. So if the amount of caffeine normally present in diet soda has a measurable but small effect when nucleation occurs via Mentos, the caffeine in your coffee powder will certainly provide the surfactant effect, more than replacing the caffeine in your decaffeinated soda, while the powder itself provides nucleation sites. Maybe the quantity is sufficient to cause foaming and flatness even if the nucleation sites aren't so obvious - nucleation is still occurring on the surface of the glass and presumably on any residue or impurities from the coffee solids. - Karenjc 21:10, 18 October 2012 (UTC)[reply]
I'll have to repeat the experiment with some well-dissolved and filtered coffee to see if it still flattens it. μηδείς (talk) 17:16, 19 October 2012 (UTC)[reply]

Periodic table of baryons

Our articles on baryons are by their nature a bit hard to comprehend... not helping is that we don't have an orderly scheme to present them all in a complete, visual array. List of baryons is a great beginning, but it doesn't give an overall impression of organization. Articles like Eightfold Way (physics) show little pieces of the elephant, but tend to leave a person more confused than ever. Now, nicer images exist - e.g. the pyramid at [31] - but even that is incomplete.

Now it occurs to me that for didactic purposes only (I'm not saying this has any deep physical meaning) we could arrange the bosons by the quark composition, as follows:

Delta                Sigma        Xi      Omega
uuu uud udd ddd | uus uds dds | uss dss | sss  (u,d,s - the [[baryon decuplet]])

                  uuc udc ddc | usc dsc | ssc
                                ucc dcc | scc  (u,d,s,c)
                                        | ccc

                                ucb dcb | scb
                                        | cbb  (u,d,s,b+c)
                                        | ccb

                  uub udb ddb | usb dsb | ssb
                                ubb dbb | sbb  (u,d,s,b)
                                        | bbb

With J=3/2 and J=1/2 states immediately above and below or side by side in a table. (This would ideally be a cute Wikitable format; I just didn't want to code such a thing when I'm just testing the breeze). Question:

  • Is this or some other "periodic table of baryons" actually in use, so it could be sourced and presented as a real concept?
  • Our list of baryons lists a number of "prime" particles which have the same quark composition and angular momentum as others. Are these misfiled - should they be moved to the section for "baryon resonance particles"?
  • Aside from the J=1/2 vs. 3/2 thing, is there a way to organize the resonance particles as part of such a "periodic table", or is there a very large or unknown potential variety of them?

Wnt (talk) 20:07, 18 October 2012 (UTC)[reply]

Is it possible to trap a photon in such way that it remains stationary ?

Hello friends, I need some help to understand photons behavior and I would like to understand the experiments of Nobel prize winners, in physics, in 2012. In everything that I could read about photons behavior, I understood that they move in light speed, they are mediator of electromagnetic forces and they don´t have rest mass because they don´t exist in stationary way. So I read this week in brasilian´s magazins that Nobel prize winners in 2012, have conducted some experiments that keep one single photon stationary ba 10 ms, in a chamber that has 25 cm. Can somebody explain a little bit what actually they did and how this affect our knowledge of photons behavior ? — Preceding unsigned comment added by Futurengineer (talkcontribs) 20:53, 18 October 2012 (UTC)[reply]


The Nobel Prize commission, on behalf of the Nobel Foundation and the Royal Swedish Academy of the Sciences, publishes a whole web-page explaining this background for the 2012 Nobel Prize in Physics. There is a press release; a "popular science" introduction to the physics; and an advanced physics brief for scientists familiar with the topic. These resources explain, in various degrees of detail, the background; the novelty of the physics discovered by the laureates, and the implications to the scientific community. We also have articles on the laureates, Serge Haroche and David J. Wineland (and in Portuguese: pt:Serge Haroche and pt:David Wineland). I believe you may be confusing photons and ions. Have you read through these resources? Any questions still unanswered? Nimur (talk) 00:08, 19 October 2012 (UTC)[reply]

Thanks Nimur, I didn´t know this site of Nobel Prize organization. I understood the way that they trapped a photon. Photon is not stationary but reflectin in two surfaces very reflective and close each other. So they send a specail athom and see in the other side if ocurred changes in phase from wave in scuh way that they can detect if therwas a photon or not in cavity. Now my question is regarding decoherence. They mentioned that they can solve problems like Schrodinger paradox and define the way that state superposition goes to just a definitive state. I don´t know how they can determine this, but my question is They can use this to observe double slit experiment and define how/when electron changes from wave to particle ? So their experiment will help to solve this strange phenomenon of quantic phisics ? — Preceding unsigned comment added by Futurengineer (talkcontribs) 02:04, 19 October 2012 (UTC)[reply]

Understanding the "live" wire in a home

Hi all, I have just removed my ceiling fan, and sticking out of the ceiling I have three disconnected wires, a black high-voltage wire, a white neutral and a green ground. With the fuse back on, I have tested the wires with a non-contact voltage sensor. Even though the hot wire is not connected to anything, the sensor still shows a AC current in it. My question: didn't we learn in high school that circuits have to be complete for current to flow (or at least be connected to ground)? Why is there an AC current in this dead-end? And why is it that even with this voltage (which has to be high enough to light up the sensor) would I not be able to light a light bulb with a single wire — I'd need to connect the other end to neutral? Thanks! — Sam 24.128.48.26 (talk) 21:54, 18 October 2012 (UTC)[reply]

Voltage is not the same thing as current. Voltage is basically a force that is capable of driving a current if you complete the circuit. Your voltage sensor, if it's the usual thing, doesn't show current. Looie496 (talk) 23:23, 18 October 2012 (UTC)[reply]
A lot of people, and a lot of books, and use the term "electromotive force," so Looie's terminology is commonplace, but it's still a little bit tricky: voltage is not a force in the same sense that a stretched spring exerts a force. Voltage is in fact a measurement of potential energy per unit charge. Nimur (talk) 23:27, 18 October 2012 (UTC)[reply]
Ok, but if there is no current in the wire than what, at it's most basic level, is causing the sensor to register something. Surly the sensor is measuring a rising and collapsing electromagnetic force, right? I assume a guassmeter would also show an EMF, right? But doesn't there have to be actual movement of electrons to cause this changing EMF? — Sam 24.128.48.26 (talk) 12:27, 19 October 2012 (UTC)[reply]
If you're working on the ceiling fan, and you didn't turn off the electric mains (by switching off a circuit-breaker, or disconnecting a fuse at the fuse-box), then you should not be working on that ceiling fan. The electricity in a household wire can be fatal. Leaving aside all other discussion about the fascinating physics associated with electricity, do not work on the wire if you aren't sure it's disconnected from the mains. Call an electrician if you aren't trained in proper procedure. In fact, the danger may arise from the same root-cause as the answer to the question: you may be completing the circuit from high-voltage to ground. If the easiest path for current to flow is through you, and not through the return wire, you can be electrocuted. For the sake of completeness, there are other reasons why your meter might show a signal, but if you aren't certain, why take the risk? Nimur (talk) 23:48, 18 October 2012 (UTC)[reply]
Thanks. I guess I figured when I said "with the fuse back on" (though I should have said circuit breaker) it was clear that I had switched the circuit breaker while I was in contact with the wires, and that my usage of the non-contact voltage tester was to be double-certain. The disconnected wires are now nine feet above the ground, so it was safe the turn the circuit breaker back on. — Sam 24.128.48.26 (talk) 12:27, 19 October 2012 (UTC)[reply]
Turn off the mains electricity for the whole house. I was working on a ceiling lighting circuit earlier this year, and gave myself a painful electric shock. I thought I had isolated the lighting circuit at the circuit breaker - but clearly there was still a significant potential difference between me and the wire I touched! AlexTiefling (talk) 09:58, 19 October 2012 (UTC)[reply]
With AC, there will be a very small AC current flow in a dead-end live wire due to capacitance. The wire is one end of an air-gap capacitor, with the other end being whatever grounded object happens to be nearest. The capacitor charges and discharges on each cycle of the AC voltage. The capacitance is very small, so the current that flows is small. The capacitance of a dead end wire is sometimes called self-capacitance, and modeled as if it did not depend on any ground being nearby (ground is assumed to be at infinity). In real situations, there will always be a ground closer than infinity. Self-capacitance is simpler to calculate, because the geometry of the grounded conductor is ignored.--Srleffler (talk) 17:42, 19 October 2012 (UTC)[reply]

Radio station reception range

About how far away could I expect to reliably receive KOKC (AM) from its transmitter location well enough to listen to University of Oklahoma football games? KOKC is a clear-channel station with technical data found here. With regards to my hometown of Salina, Kansas would I be more likely to receive the game clearly on KOKC or KGSO? I can't seem to find any stations that actually air the OU football games over their online streams, for some reason, so over the air seems to be the only way I can tune in via radio. Thanks, Ks0stm (TCGE) 22:27, 18 October 2012 (UTC)[reply]

Clear channel is an amazing thing. I've picked up WSM (AM) west of the Mississippi, as far as Albuquerque, at night-time, using no special equipment other than my Toyota's AM/FM radio. It seems a no-brainer to pick up a 50 kilowatt AM station that's a mere 250 miles distant over flat ground. Nimur (talk) 23:07, 18 October 2012 (UTC)[reply]
AM reception is very erratic, because it depends on the state of the ionosphere. Sometimes you can pick up a strong station a thousand miles away; sometimes it is noisy at fifty miles. Quite often a distant station will come through clearly for a little while and then fade out as the layers in the ionosphere shift. Looie496 (talk) 23:17, 18 October 2012 (UTC)[reply]
True, but 50 kilowatts is a lot of power; undesirable effects due to skywave multipathing, bad weather, and all that, often fall well under the noise-floor. If you look at the links on our KOKC article, one includes a map of air-checks with reliable reception ranging across most of the central United States. Nimur (talk) 23:21, 18 October 2012 (UTC)[reply]
I have to say, though, that map looks really old, and over the last 10 years or so (as long as I can reliably remember) I have noticed that AM stations seem to be becoming harder and harder to receive, even relatively locally. I don't know whether this is due to power line interference or what. I used to have practically no problem receiving KLIO in Salina (back when it was KFDI-AM), but over the years reception has gotten worse to the point where my car radio won't even pick it up reliably. Ks0stm (TCGE) 00:33, 19 October 2012 (UTC)[reply]
Well, the laws of physics haven't changed much; and the ionosphere, though it changes daily, has been closely monitored using many distinct metrics by the scientists who worry about such things. The most probable effect is that some of the AM stations you've been tuning to have reduced their transmit power as the economics of broadcast radio have changed. (Electricity isn't free, and advertising dollars on AM radio are drying up). However, KOKC and WSM, and other relics of the AM era, are currently broadcasting in the clear at fifty kilowatts, day and night, so for any specified sky conditions, you should be receiving approximately the same signal strength today as you would in 1948. KLIO, on the other hand, is authorized to broadcast only at 1 kilowatt. Nimur (talk) 00:42, 19 October 2012 (UTC)[reply]
What has changed over the years is the design of radios, particularly car radios. Up until the 1960's, radios of any quality, and car radios, were vacuum-tube based. Tube design provides high cross-modulation performance. What that is about is this: Any radio has to a cerain extent the undesirable property of cross-mixing radio signals and noise whith that of the signal/station you have tuned it to. In AM reception, the effect varies from just hearing another station in the background, to what radio people call "monkey chatter" (think of multiple Donald Ducks all taking at once) to something like white noise ie hiss. In the 1960's tube designs were replaced by bipolar transistor designs, and in car radios the tuning was based on permeability tuning - sliding magnetic cores. Bipolar transistors are not as good cross-modulation wise as tubes, but the problem was minimised to a certain extent in car radios by multiple permeability tuning circuits that reduced adjacent channel signal strength before it got to the transistors. However, in the last 10 to 15 years car radios are synthesisor based - tuned by microcircuits and not permeability tuning. This is low cost, but cross-modulation performance is very poor. In a nutshell, radios you buy today are not as good in AM noise performance as radios you used to buy.
What has also changed is the widespread use of computers, TV sets, compact flourescent lights, microwave ovens etc, that incorporate switch-mode power conversion. These devices feed back more radio frequency noise into the power wiring that older products did. An individual computer etc may not produce much interference (except close to it), but all the devices in a street each causing noise, adds up to a blanket of noise that makes AM reception harder.
Ratbone 120.145.50.173 (talk) 07:48, 19 October 2012 (UTC)[reply]

Sugar, oil, starch in plants

Are those different forms that nature found to store energy in plants? Are they equivalent replacements? And can you divide plants according to their capacity of storing sugar, oil or starch? Or can different plants in the same division implement different solutions? OsmanRF34 (talk) 23:47, 18 October 2012 (UTC)[reply]

October 19

What facts and examples should I use in a debate with a Relativity Denier?

I have been asked to 'defend Science' at a Creationist event, and the guy I'll be debating is against 'Einsteinian Relativity', or as he characterizes it, the idea that there is no universal reference frame. To summarize his position bluntly, "There IS a universal reference frame, and it is God."

I'm hardly qualified if this were any sort of scientific event, but it isn't, and looking at the guy's credentials, neither is he. What sorts of examples, facts, and topics should I review and study up on to back up... well, modern scientific thought on this?

Note: if you have any biblical quotations, those will be helpful as well, as creationists infrequently listen to logic and rationality.

Here's the 'Moderator's Introduction':

Whenever a Relativist says: “space is curved,” this merely begs the question: “Curved in relation to what?” If the Relativist says: “time slows down,” we respond: “Slows down in relation to what?” If he says that he has a “preferred frame of reference” we ask “what frame, and in reference to what?” Every proposition a Relativist utters assumes there is an absolute against which he can measure his proposition. To put it another way, the whole theory of Relativity, ironically, is based on the assumption that something is at rest. Even if he says “the speed of light is my absolute,” we respond: “the speed of light in relation to what?” And if he is someday so bold as to assume he has a “what,” we are still going to ask him “what in relation to what?” and thus require him to prove his “what” over against any other possible “whats.” If he says, “the universe is at rest” then he is once again on our side, since he has already admitted there is no difference between a rotating Earth in a fixed universe as opposed to a fixed Earth in a rotating universe. God has sprung a trap for modern man, and Relativity is its name.

I assume most of us on this site find this ridiculous, but why, specifically? I'm not asking anyone to spell out a transcript of what to say, just point me in good directions while I prepare; wiki articles to read, experiments to read up on, biblical passages to memorize, etc. — Preceding unsigned comment added by 146.63.171.193 (talk) 01:05, 19 October 2012 (UTC)[reply]

If the real issue is Creationism, it's a huge tactical error to get sucked into a debate about a different issue. It's an even bigger error if you don't actually understand that issue. Trying to win a debate by memorizing arguments you don't understand is what the bad guys do. Don't be one of the bad guys. Looie496 (talk) 01:52, 19 October 2012 (UTC)[reply]
Yes, congratulations! You have been sucked in to be the sucker they use to make whatever they want look stupid. They'll be doing their preparation too - they'll be checking the sorts of arguments put up elsewhere by people such as yourself, and will come equiped with counter arguments that worked - i.e., not counter arguments that are correct, necessarily, but arguments that experience has shown will amuse/satisfy their captive audience, and confound you.
There are a multitude of practical applications of Einstein's general theory, and especially his special theory of relativity. For example, nuclear power is one that everybody knows. E = mc2 is the core engineering equation for nuclear power. One that is less appreciated is the picture tube in TV sets (before they went to flat screens based on plasma and LCD technolgies). These worked by emitting electrons from a hot cathode at the back of the tube. Electrostatic forces are used to accelerate these electrons in a focused beam toward the screen; when they hit the screen, dissipation of their kinetic energy causes the screen to glow. Magnetics are used to make this electron beam sweep back and forth and up and down. It happens that these electrons travel at a significant fraction of the speed of light - so due to Einstein's theory, their mass increases. So the magnetic force required to sweep the beam is greater than would be expected. Picture tube engineers must understand and apply the theory in order to make tubes that work correctly. The widespread use of picture tubes and X-Ray tubes that work just as predicted by the factory engineers is a proof of Einstein's theory.
However, I'm with Looie496. What any of this has to do with creationist ideas (essentially, God created in spurt of effort every mineral and every plant and animal without using evolution) is a mystery to me.
Wickwack 124.182.48.155 (talk) 02:15, 19 October 2012 (UTC)[reply]
The introduction itself makes it obvious the person who is moderating this event does not understand relativity. You can't really debate with people who have made up their minds on a subject they haven't bothered to learn. What they really need is a physics lesson, not a debate. Someguy1221 (talk) 01:56, 19 October 2012 (UTC)[reply]
"You can't reason someone out of a position they didn't reason themselves into." -Phil Plait 67.163.109.173 (talk) 02:07, 19 October 2012 (UTC)[reply]
According to Wikiquote, Jonathan Swift said that. -- BenRG (talk) 16:41, 19 October 2012 (UTC)[reply]

Simply ask your fellow debater to express their idea in mathematical form, and use these equations to make a prediction about a physically-observable effect. For example, if they contend that there is a universal reference frame, they should be able to express the governing equations of electrodynamics in a way that predicts a variation from the documented results of the Michelson-Morley experiment. If the person can not express their ideas mathematically, or is unable to see how they relate to the question of a physically-observable fact, then inform them that they are unqualified to make assertions about the topic, and move on to discussing other interesting issues with them - issues that do not relate to empiricism and mathematical physics. Many interesting conversations are to be had with non-physicists, and sometimes even with people who hold beliefs that are contrary to empircal observations. Or, you can find a new group of people to discuss physics with, who are able to back up their assertions about the physical universe with logically-consistent observable consequences. In its present form, it seems that the debate is centered entirely on the name-label "relativity," and not actually on the mechanics of the theory that holds that name. The debate can not make progress in that form; it is little more than name-dropping and appeal to authority. One side appeals to God, and the opposing side appeals to "facts that were established by very smart and infallible physicists," but neither side actually brings any substance to the discussion. Nimur (talk) 02:21, 19 October 2012 (UTC)[reply]

Never make the mistake of thinking that because someone believes in something stupid, that they themselves are stupid. You get highly intelligent creationists! Prepare to be out argued! Wickwack 124.182.48.155 (talk) 02:25, 19 October 2012 (UTC)[reply]
You're talking at cross purposes. Moral relativity and the theory of relativity are two completely different things. You can't prove the first and final answers about the purpose of the universe or whether there is a God based on some scientific observations. And he can't predict the shift in spectral lines of a distant star based on the brightness of a "standard candle" supernova. Wnt (talk) 02:31, 19 October 2012 (UTC)[reply]
Take this opportunity to teach audience about relativity. At the end, do mention you are aware that you cannot win a debate because science is based on knowledge, and there is limit to knowledge but there is no limit to ignorance. manya (talk) 03:16, 19 October 2012 (UTC)[reply]
1)God isn't an inertial reference frame, so its ludicrous to argue God is one. 2)They need a physical model based on a classical space that accurately explains observed facts such as the Michelson-Morley experiment, relativistic Doppler relation, time dilation, length contraction and E = mc2. 3)Given a classical space, with Galilean transformations (if they know what that is) explain why if the universal vacuum speed of photons and other bosons is not invariant (the same speed in every reference frame), why it appears to be invariant. 4)If they can do the above, such a model wouldn't entail the abstract nonEuclidean geometries and maths of Minkowski space and the relativity of simultaneity. And in case any of you guys are wondering about my complete insanity, with a couple of very simple ideas, I've figured out such a model too... -Modocc (talk) 03:34, 19 October 2012 (UTC)[reply]
God and relativity are not necessarily incompatible (just like God and evolution, see theistic evolution). From the perspective of fine-tuned Universe God is the creator of physical laws and as such you can't extend scientific laws and theories, including relativity, on Him. Catholic doctrine in particular says that God exists beyond time (and of course is not affected by ageing), while relativity needs both time and space (the spacetime). Brandmeistertalk 09:53, 19 October 2012 (UTC)[reply]
That doesn't look like an impartial moderator. Would you get into a boxing ring knowing the ref would punch you too? And it looks like they'd have trouble with Newtonian relativity, never mind Einstein's Special and General theories. There are no Bible quotes that can back you up here - not because relativity is anti-Biblical, but because it deals with questions that the text was not written to answer. But if they're not Young Earth Creationists, you could do a lot worse than to argue that the six days of Creation in Genesis reflect an Iron-age approximation of the stages through which the universe went in order to produce living things. (I don't believe that myself - I think it's just the prelude to a series of moral just-so-stories - but it's not nearly as bad an argument as theirs are shaping up to be.) If they are young-earthers, then you're stuffed, because a lot of the astronomical evidence relies on accepting that there have been more than 6000 years.
I'd take the fight to them. Rather than defend your positions, try to tease out theirs. What is the speed of light in a vacuum, in their physics? If there is an absolute frame of reference in the physical world, what and where is it? If they mention God at this point, direct them to John 1:18 - "No-one has ever seen God". Attempts to express God as a physical frame of reference are bound to leave them in contradiction of scripture. But yes - Wnt is right. They have done the spectacularly ignorant thing that many such people have done, and conflated physical relativity with moral relativity.
Newton was a devout (if non-conformist) Christian, and saw the exploration of physics as an investigation of the methods and glory of God. He was certainly a moral absolutist, and yet he developed a theory of relativity that stood for 200 years, and is still the best everyday approximation of universal law. If they can't grasp the difference, they fail before you even start. AlexTiefling (talk) 09:48, 19 October 2012 (UTC)[reply]
While Alex and others have said a lot of sense, I think they are missing an important part of how you are most likely stuffed before you begin, at worst, and at best, what you really have to grapple with. This "event" - it's not just a debate between 2 opposing people with a (biased) moderator. It is relatively easy to win and argument with one or two people who have some misconceptions thru ignorance. But this "event" is I assume, to be before an audience. An audience of creationists. Creationists of all sorts maybe - ones who can think, ones who can't think. Ones who will simply not tune in to what you are saying, but will tune in whenever their side appears to land a blow. In short, its entertainment, and you are part of that entertainment, under their control. You have to win the audience, not the debator(s).
If you start talking about science or math, more than half the audience will simply tune out. If you ask them to explain with their physics what is the speed of light, they'll most likely replay with something that has nothing to do with it, but calculated to make you stop and think. Then they've won in the eyes of the audience. Wickwack 120.145.50.173 (talk) 10:35, 19 October 2012 (UTC)[reply]
I'm not against creationism, but that that is just a silly argument. Try this: since God created the universe, which includes everything (space and time), who's to say that there exists a concept such as time outside of the universe? You run into a philosophical dilema when you propose that God created the universe around Himself, that is if you agree that He was before the creation of the universe. It would be foolish to assume that God's computer runs on a Windows opperating system, just because our one does. For all we know, He could have Linux. Plasmic Physics (talk) 11:18, 19 October 2012 (UTC)[reply]
I agree with the folks who are telling you "don't even enter this debate". There is no upside for you (unless perhaps you are doing this for a bet ?). It won't even give you any useful experience of debating. If you were asked to give your opponent $100 and then convince him and his friends to give some of it back to you, would you agree ? Just tell the "moderator" that his introduction is so biased and his arguments are so absurd that you are withdrawing from the debate. Gandalf61 (talk) 11:04, 19 October 2012 (UTC)[reply]
Spot on. Of course when you do that, they may tell each other that the scientist (you) chickened out because, deep down, he/you knew your argument was never going to be any good. But that doesn't matter, if you were set up to lose anyway. Folda 120.145.5.61 (talk) 11:17, 19 October 2012 (UTC)[reply]
I agree, and this really is advanced stuff. If you really explain things you will probably lose your audience. There are things that you need to be able to understand and demonstrate mathematically. Take a observer and two people moving away in opposite directions at 25% of the speed of light, 50% of the speed of light and 100% of the speed of light - what would be observed from each point. Once understanding the theory you need to show them evidence for its correctness (Michelson–Morley experiment, clocks in plains, gravitational lenses, GPS timing), and show why it precludes there being any absolute frame of reference - which is something beyond me (after all couldn't there be some view from an extra small dimension which sees all other points as close, as some quantum theories state).
After all that they will turn round and say "well if it contradicts God being omniscient then its wrong because the bible says so". -- Q Chris (talk) 11:17, 19 October 2012 (UTC)[reply]
(Fairly ad hominem, but ask him what he thinks about Einstein. Every Relativity denier I've ever interacted with was also a fairly vicious anti-Semite and unable to hide it when talking about him, for whatever reason.) If it were me, I would just explain that Einstein's theory is a theory of gravity, it has nothing to do with morality or theology. (Einstein himself got tired of people making such connections himself and lamented that he hadn't called it the theory of invariants instead, since it is not the relativity — which is just an extension of Galilean relativity — that makes it interesting, it is the conjunction of the relativity with an invariant speed of light that comes up with funny effects.)
As a theory of gravity it has completely conformed with predictions and experiments; GPS wouldn't work if relativistic corrections weren't applied. It has absolutely no impact on whether there is or isn't a God. If you want to interpret the Big Bang as an act of Creation you are welcome to, but science isn't going to shut its ears up if it sees something that doesn't conform with your perceptions of it. This is really a textbook case of a false dilemma between science and religion — there is nothing in the equations of GR that make it even slightly against a notion of God.
I don't think you need to prove to them that GR is real or that the math is real or whatever. Kick that one down the hall to the scientists and the technologists — if GR didn't work, then various technologies based on it wouldn't work, and they clearly do work. The issue is one of the guy being confused about what he thinks the science says, and it really doesn't say that. You don't have to try and argue that God didn't exist or didn't kick off the Big Bang or something. GR has nothing to say on that point. I don't see why opposing GR is a plank of Creationism, frankly.
The moderator's intro contains many obvious errors — an actual physicist will tell you, straight up, what the reference frames are. They don't say "time slows down absolutely," they say, "time in frame A slows down relative to frame B." There's nothing inconsistent there. Personally I'd just put up the Lorentz transformation equation and the GR field equation and ask them which of the entities named is God or has any relevance to him. --Mr.98 (talk) 11:32, 19 October 2012 (UTC)[reply]
Here's an alternative approach: Attack their theology. The idea that 'God has sprung a trap for modern man' is fundamentally anti-biblical, and anti-Christian. The idea that the God of truth and love would inspire the creation of a mostly secular theory in order to deceive his suffering creation is insane. In mainstream Christian theology, God already came into the world as a human being in order to suffer and die, so that people could be freed from pain and sin. God's love is portrayed as being all-encompassing. So why on earth would he ever seek to lay a trap for us? Psalm 19 draws explicit parallels between the consistency of the physical law and the perfection of the moral law. Relativity is a consistent physical law, and experiments (including concerning the motions of the Sun, as mentioned in poetic language in the Psalm) back it up. And yet these people would rather claim that the universe is not consistent, and that God has deliberately deceived us, in order to trap us into sin and damnation.
So my 'official' advice to you - as someone with undergraduate degrees covering theology, scripture study, special relativity and formal logic - is to refuse to debate with them because they doubt the consistency and truthfulness of their own God. AlexTiefling (talk) 11:40, 19 October 2012 (UTC)[reply]
Wow, that is an awesome answer. Here is a quote "Any fool can start an argument." Plasmic Physics (talk) 11:51, 19 October 2012 (UTC)[reply]
Agreeing with Alex, you could also point out that I John 1:5 says explicitly that "God is Light", not some Universal Reference Frame. Why do they say different to their own Scripture? --TammyMoet (talk) 12:24, 19 October 2012 (UTC)[reply]
Carefull! Although the creationist said "God has sprung a trap for modern man", if you try the argument "why would a loving God set a trap for (all) mankind, they'll come back and say "God set a trap for the modern relativity scientist man, not for us faithfull creationists." Floda 120.145.5.61 (talk) 12:55, 19 October 2012 (UTC)[reply]
That sort of circular logic isn't worth engaging with. My advice is definitely to stay away, for the reasons everyone has outlined. AlexTiefling (talk) 13:03, 19 October 2012 (UTC)[reply]
  • I think the key to the argument is that, while God is universal, He cannot be the "reference frame" as the term is used in physics. To be a reference frame, you'd be using it to measure the motion of another object against. How does one measure the motion of anything against God? If your debate opponents can't answer that, you've won. --Jayron32 12:42, 19 October 2012 (UTC)[reply]
Straight to the OP's question: "Nuke them from high orbit. That's the only way to be sure."
And just as it happens, it'll demonstrate relativity in a way they can't deny... ;) — Preceding unsigned comment added by One.Ouch.Zero (talkcontribs) 13:02, 19 October 2012 (UTC)[reply]

See here and in case of objections, you refer to this page. Count Iblis (talk) 16:38, 19 October 2012 (UTC)[reply]

I agree with everybody here: don't engage in this silly "debate". Instead, take this opportunity to learn some science. I'll comment on the moderator's points, not because I think the OP should bring them up in a debate, but for the OP's own enlightenment.
1. Space is curved in the sense that if you transport a gyroscope around a closed loop, parallel to its own axis, its axis will point in a different direction than when it started. A completely uniform universe can be curved, in the same sense that a 2D bug on the surface of a balloon thinks its world is both completely uniform and positively curved.
2. Time slows down for a moving object relative to the guy observing it. That is the only sense in which time slows down; it doesn't slow down with respect to any global reference frame.
3. There is no preferred reference frame in general relativity. In special relativity, non-accelerating frames are privileged. You can tell which frames are accelerating: if you put an elevator there and get in, do you get smashed against one of the walls?
4. The speed of light is not absolute. In special relativity, it's the same in every inertial reference frame. There isn't a well-defined speed of light in an accelerating reference frame, because it depends on which direction you measure it.
5. The universe is not at rest. There is no difference between saying that the universe is rotating while Earth is not, or Earth is rotating while the universe isn't; in general relativity, every reference frame is equal. --140.180.242.9 (talk) 17:17, 19 October 2012 (UTC)[reply]

Etymology of M. manavi

Hello. When Oldfield Thomas described Miniopterus manavi in 1906, he did not explain the etymology of manavi. Does anyone have any idea what it could refer to? Thanks in advance. Leptictidium (mt) 08:46, 19 October 2012 (UTC)[reply]

An idea: from the latin "Mano"(manare, manavi, etc)-- to drip or flow [32]... so possibly it is a bat that drools a lot? Perhaps there is something "flowing" about its wings? SemanticMantis (talk) 14:06, 19 October 2012 (UTC)[reply]
Names like that are often taken from a person. I don't know who "Manav" would be, though. Looie496 (talk) 14:16, 19 October 2012 (UTC)[reply]
Yep, I thought of that too. Problem is that there appears to be no trace of anyone named "Manav" or "Manavo" who is related to Madagascar or bats. Leptictidium (mt) 14:50, 19 October 2012 (UTC)[reply]
According to this page (bottom entry), there's a Malagasy word manavy that denotes a small bat. Manavi is likely a Latinate representation of that. Deor (talk) 18:07, 19 October 2012 (UTC)[reply]

Hi all,
Apologies for creating a fuddle I should be following up myself. Looking at refs at de:Karl Emil Lischke, nl:Karl Emil Lischke, and fr:Karl Emil Lischke, I am at a loss about to migrate those to the en:wp article.
--Shirt58 (talk) 12:19, 19 October 2012 (UTC)[reply]

Edit: Still can't find refs in English.--Shirt58 (talk) 13:21, 19 October 2012 (UTC)[reply]
You don't have to have English refs if none are easily available. I suggest you use the French refs on the grounds that they are (possibly) the easiest for native English speakers to understand. There is discussion about this theme at the Village Pump. Richard Avery (talk) 14:02, 19 October 2012 (UTC)[reply]
There are no French refs; the French article's only source is the German article. Isn't one of the refs on the German article in English? --140.180.242.9 (talk) 16:50, 19 October 2012 (UTC)[reply]